2024-07-12
Solutions to Principles of Quantum Mechanics
00

目录

Chapter 1 Mathemtical Introduction
1.1 Linear Vector Spaces: Basics
1.2 Inner Product Spaces
1.3 Dual Spaces and the Dirac Notation
1.4 Subspaces
1.5 Linear Operators
1.6 Matrix Elements of Linear Operators
1.7 Active and Passive Transformations
1.8 The Eigenvalue Problem
1.9 Functions of Operators and Related Concepts
1.10 Generalization to Infinite Dimensions

Chapter 1 Mathemtical Introduction

1.1 Linear Vector Spaces: Basics

Exercise 1.1.1 Verify these claims. For the first consider 0+0|0\rangle+|0^{\prime}\rangle and use the advertised properties of the two null vectors in turn. For the second start with 0=(0+1)V+V|0\rangle=(0+1)|V\rangle+|-V\rangle. For the third, begin with V+(V)=0V=0|V\rangle+(-|V\rangle)=0|V\rangle=|0\rangle. For the last, let W|W\rangle also satisfy V+W=0|V\rangle+|W\rangle=|0\rangle. Since 0|0\rangle is unique, this means V+W=V+V|V\rangle+|W\rangle=|V\rangle+|-V\rangle. Take it from here.

Solution.

(1) 0|0\rangle is unique.

Proof. For an arbitrary state ket V|V\rangle,

(a) V+0=V|V\rangle+|0\rangle=|V\rangle

(b) V+0=V|V\rangle+|0^{\prime}\rangle=|V\rangle

Set V=0|V\rangle=|0^{\prime}\rangle in (a) \Rightarrow 0+0=0|0^{\prime}\rangle+|0\rangle=|0^{\prime}\rangle;

Set V=0|V\rangle=|0\rangle in (ii) \Rightarrow 0+0=0|0\rangle+|0^{\prime}\rangle=|0\rangle;

Therefore, by commutativity of vector addition, we have

0=0+0=0+0=0|0^{\prime}\rangle=|0^{\prime}\rangle+|0\rangle=|0\rangle+|0^{\prime}\rangle=|0\rangle
 ~\tag*{$\square$}

(2) 0V=00|V\rangle=|0\rangle

Proof. We have

1V=(1+0)V=1V+0V1|V\rangle=(1+0)|V\rangle=1|V\rangle+0|V\rangle

where 1V=V1|V\rangle=|V\rangle. Therefore

V=V+0V|V\rangle=|V\rangle+0|V\rangle

Since V|V\rangle is arbitrary here, compared with the definition of 0|0\rangle, which is V+0=V|V\rangle+|0\rangle=|V\rangle for any V|V\rangle, plus 0|0\rangle is unique, we can conclude that

0V=0.0|V\rangle=|0\rangle.
 ~\tag*{$\square$}

(3) V=V|-V\rangle=-|V\rangle

Proof. For arbitrary V|V\rangle,

V+(V)=0V=0.|V\rangle+(-|V\rangle)=0|V\rangle=|0\rangle.

By definition of V|-V\rangle, which is V+V=0|V\rangle+|-V\rangle=0, we can conclude that

V=V.|-V\rangle=-|V\rangle.
 ~\tag*{$\square$}

(4) V|-V\rangle is the unique addtive inverse of V|V\rangle.

Proof. Suppose there exists another vector W|W\rangle, satisfying W=V|W\rangle=-|V\rangle, then

V+W=VV=(11)V=0V=0\begin{aligned} |V\rangle+|W\rangle&=|V\rangle-|V\rangle\\ &=(1-1)|V\rangle\\ &=0|V\rangle\\ &=|0\rangle \end{aligned}

Add V|-V\rangle on the both sides, we have

V+W+V=0+VW+(V+V)=VW+0=V\begin{aligned} |V\rangle+|W\rangle+|-V\rangle&=|0\rangle+|-V\rangle\\ |W\rangle+(|V\rangle+|-V\rangle)&=|-V\rangle\\ |W\rangle+|0\rangle&=|-V\rangle \end{aligned}

Therefore,

W=V|W\rangle=|-V\rangle
 ~\tag*{$\square$}
 ~\tag*{$\blacksquare$}

Exercise 1.1.2 Consider the set of all entities of the form (a,b,c)(a, b, c) where the entries are real numbers. Addition and scalar multiplication are defined as follows:

(a,b,c)+(d,e,f)=(a+d,b+e,c+f)α(a,b,c)=(αa,αb,αc).\begin{aligned} (a, b, c)+(d, e, f)&=(a+d, b+e, c+f) \\ \alpha(a, b, c)&=(\alpha a, \alpha b, \alpha c). \end{aligned}

Write down the null vector and inverse of (a,b,c)(a, b, c). Show that vectors of the form (a,b,1)(a, b, 1) do not form a vector space.

Solution.

  • Null vector of (a,b,c)(a,b,c): By definition, for any V|V\rangle,
V+0=V.|V\rangle+|0\rangle=|V\rangle.

Set 0=(a0,b0,c0)|0\rangle=(a_{0},b_{0},c_{0}), V=(a,b,c)|V\rangle=(a,b,c), where a,b,ca,b,c are arbitrary numbers. Then

0+V=(a0,b0,c0)+(a,b,c)=(a0+a,b0+b,c0+c)=V=(a,b,c) \begin{aligned} |0\rangle+|V\rangle&=(a_{0},b_{0},c_{0})+(a,b,c)\\ &=(a_{0}+a,b_{0}+b,c_{0}+c)\\ &=|V\rangle\\ &=(a,b,c) \end{aligned}
{a0+a=ab0+b=bc0+c=c{a0=0b0=0c0=0 \Rightarrow\quad\left\{ \begin{aligned} a_{0}+a=a\\ b_{0}+b=b\\ c_{0}+c=c \end{aligned} \right.\quad \Rightarrow\quad\left\{ \begin{aligned} a_{0}=0\\ b_{0}=0\\ c_{0}=0 \end{aligned} \right.

Therefore, the null vector of (a,b,c)(a,b,c) is (0,0,0)(0,0,0).

  • Inverse vector of (a,b,c)(a,b,c): Suppose the inverse vector of (a,b,c)(a,b,c) is (aˉ,bˉ,cˉ)(\bar{a},\bar{b},\bar{c}). By definition,
(a,b,c)+(aˉ,bˉ,cˉ)=0=(0,0,0)(a,b,c)+(\bar{a},\bar{b},\bar{c})=|0\rangle=(0,0,0)
(a+aˉ,b+bˉ,c+cˉ)=(0,0,0) (a+\bar{a},b+\bar{b},c+\bar{c})=(0,0,0)
{a+aˉ=0b+bˉ=0c+cˉ=0{aˉ=abˉ=bcˉ=c \Rightarrow\quad\left\{ \begin{aligned} a+\bar{a}=0\\ b+\bar{b}=0\\ c+\bar{c}=0 \end{aligned} \right.\quad \Rightarrow\quad\left\{ \begin{aligned} \bar{a}=-a\\ \bar{b}=-b\\ \bar{c}=-c \end{aligned} \right.

Therefore, the inverse vector of (a,b,c)(a,b,c) is (a,b,c)(-a,-b,-c).

  • {(a,b,1)}\{(a,b,1)\} does not form a vector space since

(a) It violates the closure under addition, i.e.

(a1,b1,1)+(a2,b2,1)=(a1+a2,b1+b2,2){(a,b,1)}.(a_{1},b_{1},1)+(a_{2},b_{2},1)=(a_{1}+a_{2},b_{1}+b_{2},2)\notin \{(a,b,1)\}.

(b) It violates the closure under scalar multiplication, i.e.

ω(a1,b1,1)=(ωa1,ωb1,ω){(a,b,1)}\omega(a_{1},b_{1},1)=(\omega a_{1},\omega b_{1},\omega)\notin\{(a,b,1)\}

as long as ω1\omega\neq 1. (c) There is no null vector, i.e.

(0,0,0){(a,b,1)}. (0,0,0)\notin \{(a,b,1)\}.

(d) The inverse does not exist, i.e.

(a,b,1){(a,b,1)} (-a,-b,-1)\notin \{(a,b,1)\}
 ~\tag*{$\blacksquare$}

Exercise 1.1.3 Do functions that vanish at the end points x=0x=0 and x=Lx=L form a vector space? How about periodic functions obeying f(0)=f(L)f(0)=f(L) ? How about functions that obey f(0)=4f(0)=4 ? If the functions do not qualify, list the things that go wrong.

Solution.

(1) {f(x)}\{f(x)\}, f(0)=f(L)=0f(0)=f(L)=0, form a vector space.

(2) {f(x)}\{f(x)\}, periodic functions obeying f(0)=f(L)f(0)=f(L), form a vector space. If you want to prove the property of closure in this problem, please mention that f(x)+g(x)f(x)+g(x) and αf(x)\alpha f(x) are also periodic functions. That is, f(0)+g(0)=f(L)+g(L)f(0)+g(0)=f(L)+g(L), αf(0)=αf(L)\alpha f(0)=\alpha f(L).

(3) {f(x)}\{f(x)\}, f(0)=4f(0)=4, do not form a vector space, since

(a) If g(x),h(x){f(x)}g(x), h(x)\in \{f(x)\}, then g(x)+h(x){f(x)}g(x)+h(x)\notin \{f(x)\}, since g(0)+h(0)=84g(0)+h(0)=8\neq 4.

(b) If g(x){f(x)}g(x)\in\{f(x)\}, then λg(x){f(x)}\lambda g(x)\notin \{f(x)\}, since λg(0)=4λ4\lambda g(0)=4\lambda\neq 4, as long as λ1\lambda\neq 1.

(c) No null vector. g(x)0{f(x)}g(x)\equiv 0\notin \{f(x)\}, since g(0)=04g(0)=0\neq 4.

(d) If g(x){f(x)}g(x)\in \{f(x)\}, then the inverse g(x){f(x)}-g(x)\notin \{f(x)\}, since g(0)=44-g(0)=-4\neq 4.

 ~\tag*{$\blacksquare$}

Exercise 1.1.4 Consider three elements from the vector space of real 2×22 \times 2 matrices:

1=(0100)2=(1101)3=(2102) |1\rangle=\begin{pmatrix} 0 & 1 \\ 0 & 0 \end{pmatrix} \quad|2\rangle=\begin{pmatrix} 1 & 1 \\ 0 & 1 \end{pmatrix} \quad|3\rangle=\begin{pmatrix} -2 & -1 \\ 0 & -2 \end{pmatrix}

Are they linearly independent? Support your answer with details. (Notice we are calling these matrices vectors and using kets to represent them to emphasize their role as elements of a vector space.)

Solution. Suppose α11+α22+α33=0\alpha_{1}|1\rangle+\alpha_{2}|2\rangle+\alpha_{3}|3\rangle=0. We have

(0α1+1α2+(2)α31α1+1α2+(1)α30α1+0α2+0α30α1+1α2+(2)α3)=(0000) \begin{pmatrix} 0\cdot \alpha_{1}+1\cdot \alpha_{2}+(-2)\cdot \alpha_{3}&1\cdot \alpha_{1}+1\cdot \alpha_{2}+(-1)\cdot \alpha_{3}\\ 0\cdot \alpha_{1}+0\cdot \alpha_{2}+0\cdot \alpha_{3}&0\cdot \alpha_{1}+1\cdot \alpha_{2}+(-2)\cdot \alpha_{3} \end{pmatrix}= \begin{pmatrix} 0&0\\ 0&0 \end{pmatrix}
{α22α3=0α1+α2α3=0 \Rightarrow\quad\left\{ \begin{aligned} \alpha_{2}-2\alpha_{3}=0\\ \alpha_{1}+\alpha_{2}-\alpha_{3}=0 \end{aligned}\right.
{α1=α3α2=2α3 \Rightarrow\quad \left\{ \begin{aligned} \alpha_{1}=-\alpha_{3}\\ \alpha_{2}=2\alpha_{3} \end{aligned}\right.

It is not necessary for α1,α2\alpha_{1}, \alpha_{2} and α3\alpha_{3} to be 00 together. Therefore, 1|1\rangle, 2|2\rangle and 3|3\rangle are linearly dependent.

 ~\tag*{$\blacksquare$}

Exercise 1.1.5 Show that the following row vectors are linearly dependent: (1,1,0)(1,1,0), (1,0,1)(1,0,1), and (3,2,1)(3,2,1). Show the opposite for (1,1,0),(1,0,1)(1,1,0),(1,0,1), and (0,1,1)(0,1,1).

Solution. Suppose α1(1,1,0)+α2(1,0,1)+α3(3,2,1)=0\alpha_{1}(1,1,0)+\alpha_{2}(1,0,1)+\alpha_{3}(3,2,1)=0. Then

{α1+α2+3α3=0α1+2α3=0α2+α3=0{α1=2α3α2=α3 \left\{ \begin{aligned} \alpha_{1}+\alpha_{2}+3\alpha_{3}=0\\ \alpha_{1}+2\alpha_{3}=0\\ \alpha_{2}+\alpha_{3}=0 \end{aligned}\right.\quad\Rightarrow\quad \left\{ \begin{aligned} \alpha_{1}=-2\alpha_{3}\\ \alpha_{2}=-\alpha_{3} \end{aligned}\right.

When α30\alpha_{3}\neq 0. α1,α2,α3\alpha_{1}, \alpha_{2}, \alpha_{3} can have non-zero values. Therefore, (1,1,0)(1,1,0), (1,0,1)(1,0,1), (3,2,1)(3,2,1) are linearly dependent.

Suppose α1(1,1,0)+α2(1,0,1)+α3(0,1,1)=0\alpha_{1}(1,1,0)+\alpha_{2}(1,0,1)+\alpha_{3}(0,1,1)=0. Then

{α1+α2=0α1+α3=0α2+α3=0{α1=0α2=0α3=0 is the only solution. \left\{ \begin{aligned} \alpha_{1}+\alpha_{2}=0\\ \alpha_{1}+\alpha_{3}=0\\ \alpha_{2}+\alpha_{3}=0 \end{aligned}\right.\quad\Rightarrow\quad \left\{ \begin{aligned} \alpha_{1}=0\\ \alpha_{2}=0\\ \alpha_{3}=0 \end{aligned}\right.~\text{is the only solution.}

Therefore, (1,1,0)(1,1,0), (1,0,1)(1,0,1), (0,1,1)(0,1,1) are linearly independent.

 ~\tag*{$\blacksquare$}

1.2 Inner Product Spaces

1.3 Dual Spaces and the Dirac Notation

Exercise 1.3.1 Form an orthonormal basis in two dimensions starting with A=3i+4j\vec{A}=3 \vec{i}+4 \vec{j} and B=2i6j\vec{B}=2 \vec{i}-6 \vec{j}. Can you generate another orthonormal basis starting with these two vectors? If so, produce another.

Solution. Using Gram-Schmidt process here, starting from A\vec{A}.

e1=AA=3i+4j32+42=35i+45j \vec{e}_{1}=\frac{\vec{A}}{|\vec{A}|}=\frac{3\vec{i}+4\vec{j}}{\sqrt{3^{2}+4^{2}}}=\frac{3}{5}\vec{i}+\frac{4}{5}\vec{j}
e2=B(Be1)e1=(2i6j)(65245)(35i+45j)=(2+5425)i+(6+7225)j=10425i7825j \begin{aligned} \vec{e}_{2}^{\prime}&=\vec{B}-(\vec{B}\cdot \vec{e}_{1})\vec{e}_{1}\\ &=(2\vec{i}-6\vec{j})-\left(\frac{6}{5}-\frac{24}{5}\right)\left(\frac{3}{5}\vec{i}+\frac{4}{5}\vec{j}\right)\\ &=\left(2+\frac{54}{25}\right)\vec{i}+\left(-6+\frac{72}{25}\right)\vec{j}\\ &=\frac{104}{25}\vec{i}-\frac{78}{25}\vec{j} \end{aligned}
e2=e2e2=10425i7825j(10425)2+(7825)2=45i35j \vec{e}_{2}=\frac{\vec{e}_{2}^{\prime}}{|\vec{e}_{2}^{\prime}|}=\frac{\frac{104}{25}\vec{i}-\frac{78}{25}\vec{j}}{\sqrt{\left(\frac{104}{25}\right)^{2}+\left(\frac{78}{25}\right)^{2}}}=\frac{4}{5}\vec{i}-\frac{3}{5}\vec{j}

Therefore, the new basis is e1=35i+45j\vec{e}_{1}=\frac{3}{5}\vec{i}+\frac{4}{5}\vec{j}, e2=45i35j\vec{e}_{2}=\frac{4}{5}\vec{i}-\frac{3}{5}\vec{j}.

 ~\tag*{$\blacksquare$}

Exercise 1.3.2 Show how to go from the basis

I=(300)II=(012)III=(025) |I\rangle=\begin{pmatrix} 3 \\ 0 \\ 0 \end{pmatrix} \quad|I I\rangle= \begin{pmatrix} 0 \\ 1 \\ 2 \end{pmatrix}\quad|I II\rangle=\begin{pmatrix} 0 \\ 2 \\ 5 \end{pmatrix}

to the orthonormal basis

1=(100)2=(01/52/5)3=(02/51/5) |1\rangle=\begin{pmatrix} 1 \\ 0 \\ 0 \end{pmatrix} \quad|2\rangle=\begin{pmatrix} 0 \\ 1 / \sqrt{5} \\ 2 / \sqrt{5} \end{pmatrix}\quad|3\rangle=\begin{pmatrix} 0 \\ -2 / \sqrt{5} \\ 1 / \sqrt{5} \end{pmatrix}

Solution.

1=III=13(300)=(100)2=II(1II)1=(012)0(100)=(012)2=222=15(012)=(01/52/5)3=III(1II)1(2III)2=(025)0(100)(0+25+105)(01/52/5)=(025)(012/524/5)=(02/51/5)3=333=(02/51/5) \begin{aligned} |1\rangle&=\frac{|I\rangle}{\sqrt{\langle I \mid I\rangle}}=\frac{1}{3}\begin{pmatrix} 3 \\ 0 \\ 0 \end{pmatrix}=\begin{pmatrix} 1 \\ 0 \\ 0 \end{pmatrix}\\ \left|2^{\prime}\right\rangle & =|II\rangle-(\langle 1 \mid II\rangle)|1\rangle \\ &=\begin{pmatrix} 0 \\ 1 \\ 2 \end{pmatrix}-0 \cdot\begin{pmatrix} 1 \\ 0 \\ 0 \end{pmatrix} \\ & =\begin{pmatrix} 0 \\ 1 \\ 2 \end{pmatrix}\\ |2\rangle&=\frac{\left|2^{\prime}\right\rangle}{\sqrt{\left\langle 2^{\prime} \mid 2^{\prime}\right\rangle}}=\frac{1}{\sqrt{5}}\begin{pmatrix} 0 \\ 1 \\ 2 \end{pmatrix}=\begin{pmatrix} 0 \\ 1/\sqrt{5} \\ 2/\sqrt{5} \end{pmatrix}\\ \left|3^{\prime}\right\rangle & =\mid III \rangle-(\langle 1\mid II \rangle)|1\rangle-(\langle 2\mid III\rangle)|2\rangle \\ & =\begin{pmatrix} 0 \\ 2 \\ 5 \end{pmatrix}-0 \cdot\begin{pmatrix} 1 \\ 0 \\ 0 \end{pmatrix}-\left(0+\frac{2}{\sqrt{5}}+\frac{10}{\sqrt{5}}\right)\begin{pmatrix} 0 \\ 1/\sqrt{5} \\ 2/\sqrt{5} \end{pmatrix} \\ & =\begin{pmatrix} 0 \\ 2 \\ 5 \end{pmatrix} -\begin{pmatrix} 0 \\ 12/5 \\ 24/5 \end{pmatrix} \\ & =\begin{pmatrix} 0 \\ -2/5 \\ 1/5 \end{pmatrix}\\ |3\rangle&=\frac{\left|3^{\prime}\right\rangle}{\sqrt{\left\langle 3^{\prime} \mid 3^{\prime}\right\rangle}}=\begin{pmatrix} 0 \\ -2 / \sqrt{5} \\ 1 / \sqrt{5} \end{pmatrix} \end{aligned}
 ~\tag*{$\blacksquare$}

Exercise 1.3.3 When will this equality VV=WVVWW2\langle V \mid V\rangle=\frac{\langle W \mid V\rangle\langle V \mid W\rangle}{|W|^2} be satisfied? Does this agree with your experience with arrows?

Solution. When V=CW|V\rangle=C|W\rangle, we have

VV=C2WW=C2W2 \langle V\mid V\rangle=|C|^{2}\langle W\mid W\rangle=|C|^{2}|W|^{2}

Also

WVVW=(CWW)(CWW)=C2WWWW=C2W4 \begin{aligned} \langle W\mid V\rangle\langle V\mid W\rangle&=(C\langle W\mid W\rangle)(C^{\star}\langle W\mid W\rangle)\\ &=|C|^{2}\langle W\mid W\rangle\langle W\mid W\rangle\\ &=|C|^{2}|W|^{4} \end{aligned}

Hence,

VV=WVVWW2 \langle V\mid V\rangle=\frac{\langle W\mid V\rangle\langle V\mid W\rangle}{|W|^{2}}

When two arrows are parallel or anti-parallel with each other, the square of their inner product equals to the product of their norms.

 ~\tag*{$\blacksquare$}

Exercise 1.3.4 Prove the triangle inequality starting with V+W2|V+W|^{2}. You must use ReVWVW\mathrm{Re}\langle V|W\rangle\leqslant |\langle V|W\rangle| and the Schwarz inequality. Show that the final inequality becomes an equality only if V=aW|V\rangle=a|W\rangle where aa is a real positive scalar.

Solution. ReVWVWVW\mathrm{Re}\langle V\mid W\rangle\leqslant |\langle V\mid W\rangle|\leqslant |V||W|

ReVW2VW \Rightarrow\quad \mathrm{Re}\langle V\mid W\rangle\leqslant 2|V||W|

Add V2+W2|V|^{2}+|W|^{2} to both sides of the inequality above, we have

VV+2ReVW+WWV2+W2+2VW \langle V\mid V\rangle+2\mathrm{Re}\langle V\mid W\rangle+\langle W\mid W\rangle\leqslant |V|^{2}+|W|^{2}+2|V||W|
LHS=VV+VW+WV+WW=V+WV+W=V+W2RHS=(V+W)2 \begin{aligned} \text{LHS}&=\langle V\mid V\rangle+\langle V\mid W\rangle+\langle W\mid V\rangle+\langle W\mid W\rangle\\ &=\langle V+W\mid V+W\rangle\\ &=|V+W|^{2}\\ \text{RHS}&=(|V|+|W|)^{2} \end{aligned}

Therefore,

V+W2(V+W)2 |V+W|^{2}\leqslant (|V|+|W|)^{2}
V+WV+W(the triangular inequality) \Rightarrow\quad |V+W|\leqslant |V|+|W|\tag{the triangular inequality}

Attention: We are supposed to prove the equality holds only if V=αW|V\rangle=\alpha|W\rangle, where α\alpha is a real number.

The equality holds only if the following two equalities hold:

(a) VW=VW\langle V\mid W\rangle=|V||W|

(b) ReVW=VW\mathrm{Re}\langle V\mid W\rangle=|\langle V\mid W\rangle|

From the proof process of the Schwarz inequality in Shankar, we know that equality (a) holds only if

Z=VWVW2W=0 |Z\rangle=|V\rangle-\frac{\langle W\mid V\rangle}{|W|^{2}}|W\rangle=0

which means V|V\rangle must be able to expressed as αW\alpha |W\rangle, where α\alpha is a number. To prove that α\alpha must be real, we substitude V=αW|V\rangle=\alpha |W\rangle into equality (b) above.

VW=αVV \langle V\mid W\rangle=\alpha^{\star}\langle V\mid V\rangle

To satisfy equality (b), VW\langle V\mid W\rangle must be real. Since VV\langle V\mid V\rangle is real, α\alpha^{\star} must be real. Therefore, α\alpha must be a real number.

 ~\tag*{$\blacksquare$}

1.4 Subspaces

Exercise 1.4.1 In a space Vn\mathbb{V}^n, prove that the set of all vectors {V1,V2,}\left\{\left|V_{\perp}^1\right\rangle,\left|V_{\perp}^2\right\rangle, \ldots\right\}, orthogonal to any V0|V\rangle \neq |0\rangle, form a subspace Vn1\mathbb{V}^{n-1}.

Solution. Given a vector space Vn\mathbb{V}^{n}, one can start with an arbitrary vector V0|V\rangle\neq 0 and construct n1n-1 other vectors orthogonal to this V|V\rangle through Gram-Schmidt process. Since these n1n-1 vectors are linear independent, they span a Vn1\mathbb{V}^{n-1} subspace. Now we prove that this subspace Vn1\mathbb{V}^{n-1} is the set of all vectors orthogonal to V|V\rangle.

(1) Every vector in Vn1\mathbb{V}^{n-1} is orthogonal to V|V\rangle: Since every vector in Vn1\mathbb{V}^{n-1} can be expressed as a linear combination of the n1n-1 vectors we constructed above V=i=1n1αiVi|V_{\perp}\rangle=\sum\limits_{i=1}^{n-1}\alpha_{i}|V_{i}\rangle, and VV=i=1n1αiVVi=0\langle V\mid V_{\perp}\rangle=\sum\limits_{i=1}^{n-1}\alpha_{i}\langle V\mid V_{i}\rangle=0, because VVi=0\langle V\mid V_{i}\rangle=0 for each ViV_{i}.

(2) Every vector in Vn\mathbb{V}^{n} but outside Vn1\mathbb{V}^{n-1} is not orthogonal to V|V\rangle: Since this kind of vectors can be expressed as W=αV+i=1n1αiVi|W\rangle=\alpha|V\rangle+\sum\limits_{i=1}^{n-1}\alpha_{i}|V_{i}\rangle, where α0\alpha\neq 0. Therefore VW=αVV=α0\langle V\mid W\rangle=\alpha\langle V\mid V\rangle=\alpha\neq 0.

 ~\tag*{$\blacksquare$}

Exercise 1.4.2 Suppose V1n1\mathbb{V}_1^{n_1} and V2n2\mathbb{V}_2^{n_2} are two subspaces such that any element of V1\mathbb{V}_1 is orthogonal to any element of V2\mathbb{V}_2. Show that the dimensionality of V1V2\mathbb{V}_1 \oplus \mathbb{V}_2 is n1+n2n_1+n_2. (Hint: Theorem 4.)

Solution. Since V1n1\mathbb{V}_{1}^{n_{1}} and V2n2\mathbb{V}_{2}^{n_{2}} are two subspace orthogonal to each other, we can take the n1n_{1} basis vectors of V1n1\mathbb{V}_{1}^{n_{1}} and the n2n_{2} basis vectors of V2n2\mathbb{V}_{2}^{n_{2}}, and put them together. Because these n1+n2n_{1}+n_{2} vectors are orthogonal to each other, they can span a Vn1+n2\mathbb{V}^{n_{1}+n_{2}} subspace. We now prove that this Vn1+n2\mathbb{V}^{n_{1}+n_{2}} is nothing but the V1V2\mathbb{V}_{1}\oplus \mathbb{V}_{2}.

(1) Every vector in Vn1+n2\mathbb{V}^{n_{1}+n_{2}} is in V1V2\mathbb{V}_{1}\oplus \mathbb{V}_{2}:

Since each vector in Vn1+n2\mathbb{V}^{n_{1}+n_{2}} can be expressed as U=i=1n1αiVi+j=1n2βjWj|U\rangle=\sum\limits_{i=1}^{n_{1}}\alpha_{i}|V_{i}\rangle+\sum\limits_{j=1}^{n_{2}}\beta_{j}|W_{j}\rangle, where {Vi}\{|V_{i}\rangle\}, {Wj}\{|W_{j}\rangle\} are basis vectors of V1\mathbb{V}_{1} and V2\mathbb{V}_{2} respectively. Notice that i=1n1αiVi\sum\limits_{i=1}^{n_{1}}\alpha_{i}|V_{i}\rangle is a vector in V1\mathbb{V}_{1}, and j=1n2bjWj\sum\limits_{j=1}^{n_{2}}b_{j}|W_{j}\rangle is a vector in V2\mathbb{V}_{2}. Therefore U|U\rangle can be expressed as a combination of vectors from V1\mathbb{V}_{1} and V2\mathbb{V}_{2}. According to the definition of V1V2\mathbb{V}_{1}\oplus \mathbb{V}_{2} (Definition 12 in Shankar), UV1V2|U\rangle\in \mathbb{V}_{1}\oplus \mathbb{V}_{2}.

(2) Every vector in V1V2\mathbb{V}_{1}\oplus \mathbb{V}_{2} is in Vn1+n2\mathbb{V}^{n_{1}+n_{2}}:

Every vector in V1V2\mathbb{V}_{1}\oplus \mathbb{V}_{2} can be expressed as Z=C1Z1+C2Z2|Z\rangle=C_{1}|Z_{1}\rangle+C_{2}|Z_{2}\rangle, where Z1V1n1|Z_{1}\rangle\in\mathbb{V}_{1}^{n_{1}}, |Z2V2n2Z_{2}\rangle\in \mathbb{V}_{2}^{n_{2}}. Therefore, Z1=i=1n1piVi|Z_{1}\rangle=\sum\limits_{i=1}^{n_{1}}p_{i}|V_{i}\rangle, and Z2=j=1n2qjWj|Z_{2}\rangle=\sum\limits_{j=1}^{n_{2}}q_{j}|W_{j}\rangle. Thus, Z=i=1n1C1piVi+j=1n2C2qjWj|Z\rangle=\sum\limits_{i=1}^{n_{1}}C_{1}p_{i}|V_{i}\rangle+\sum\limits_{j=1}^{n_{2}}C_{2}q_{j}|W_{j}\rangle, which lies in Vn1+n2\mathbb{V}^{n_{1}+n_{2}}.

Therefore, by theorem 4 in Shankar, there are n1+n2n_{1}+n_{2} orthogonal vectors in V1V2\mathbb{V}_{1}\oplus \mathbb{V}_{2}, so the dimension of V1V2\mathbb{V}_{1}\oplus \mathbb{V}_{2} is n1+n2n_{1}+n_{2}.

 ~\tag*{$\blacksquare$}

1.5 Linear Operators

1.6 Matrix Elements of Linear Operators

Exercise 1.6.1 An operator Ω\Omega is given by the matrix

(001100010) \begin{pmatrix} 0&0&1\\ 1&0&0\\ 0&1&0 \end{pmatrix}

What is its action?

Solution. To see Ω\Omega's action, let's act on basis vectors:

Ω1=(001100010)(100)=(010)=2Ω2=(001100010)(010)=(001)=3Ω3=(001100010)(001)=(100)=1 \begin{aligned} \Omega|1\rangle&=\begin{pmatrix} 0&0&1\\ 1&0&0\\ 0&1&0 \end{pmatrix}\begin{pmatrix} 1\\0\\0 \end{pmatrix}=\begin{pmatrix} 0\\1\\0 \end{pmatrix}=|2\rangle\\ \Omega|2\rangle&=\begin{pmatrix} 0&0&1\\ 1&0&0\\ 0&1&0 \end{pmatrix}\begin{pmatrix} 0\\1\\0 \end{pmatrix}=\begin{pmatrix} 0\\0\\1 \end{pmatrix}=|3\rangle\\ \Omega|3\rangle&=\begin{pmatrix} 0&0&1\\ 1&0&0\\ 0&1&0 \end{pmatrix}\begin{pmatrix} 0\\0\\1 \end{pmatrix}=\begin{pmatrix} 1\\0\\0 \end{pmatrix}=|1\rangle \end{aligned}

This is a cyclic permutation of the three basis vectors.

It is equivalent to rotation of the coordinate axis along (1,1,1)(1,1,1) by 2π3\frac{2\pi}{3}.

 ~\tag*{$\blacksquare$}

Exercise 1.6.2 Given Ω\Omega and Λ\Lambda are Hermitian what can you say about (1) ΩΛ\Omega \Lambda; (2) ΩΛ+ΛΩ\Omega \Lambda+\Lambda \Omega; (3) [Ω,Λ][\Omega, \Lambda]; and (4) i[Ω,Λ]\mathrm{i}[\Omega, \Lambda] ?

Solution.

(1) Not Hermitian: (ΩΛ)=ΛΩ=ΛΩΩΛ(\Omega\Lambda)^{\dagger}=\Lambda^{\dagger}\Omega^{\dagger}=\Lambda\Omega\neq \Omega\Lambda

(2) Hermitian:

(ΩΛ+ΛΩ)=(ΩΛ)+(ΛΩ)=ΛΩ+ΩΛ=ΛΩ+ΩΛ=ΩΛ+ΛΩ \begin{aligned} (\Omega\Lambda+\Lambda\Omega)^{\dagger}&=(\Omega\Lambda)^{\dagger}+(\Lambda\Omega)^{\dagger}\\ &=\Lambda^{\dagger}\Omega^{\dagger}+\Omega^{\dagger}\Lambda^{\dagger}\\ &=\Lambda\Omega+\Omega\Lambda\\ &=\Omega\Lambda+\Lambda\Omega \end{aligned}

(3) Anti-Hermitian:

[Ω,Λ]=(ΩΛΛΩ)=(ΩΛ)(ΛΩ)=ΛΩΩΛ=ΛΩΩΛ=(ΩΛΛΩ)=[Ω,Λ] \begin{aligned} \left[\Omega,\Lambda\right]^{\dagger}&=(\Omega\Lambda-\Lambda\Omega)^{\dagger}\\ &=(\Omega\Lambda)^{\dagger}-(\Lambda\Omega)^{\dagger}\\ &=\Lambda^{\dagger}\Omega^{\dagger}-\Omega^{\dagger}\Lambda^{\dagger}\\ &=\Lambda\Omega-\Omega\Lambda\\ &=-(\Omega\Lambda-\Lambda\Omega)\\ &=-[\Omega,\Lambda] \end{aligned}

(4) Hermitian:

(i[Ω,Λ])=i[Ω,Λ]=i([Ω,Λ])=[Ω,Λ] \begin{aligned} (\mathrm{i}\left[\Omega,\Lambda\right])^{\dagger}&=-\mathrm{i}\left[\Omega,\Lambda\right]^{\dagger}\\ &=-\mathrm{i}\cdot (-\left[\Omega,\Lambda\right])\\ &=[\Omega,\Lambda] \end{aligned}
 ~\tag*{$\blacksquare$}

Exercise 1.6.3 Show that a product of unitary operator is unitary.

Solution. Suppose U1,U2U_{1}, U_{2} are unitary, which means that

U1U1=I=U2U2 U_{1}^{\dagger}U_{1}=\mathbb{I}=U_{2}^{\dagger}U_{2}

Therefore,

(U1U2)(U1U2)=U2U1U1U2=U2(U1U1)U2=U2IU2=U2U2=I \begin{aligned} (U_{1}U_{2})^{\dagger}(U_{1}U_{2})&=U_{2}^{\dagger}U_{1}^{\dagger}U_{1}U_{2}\\ &=U_{2}^{\dagger}(U_{1}^{\dagger}U_{1})U_{2}\\ &=U_{2}^{\dagger}\mathbb{I}\,U_{2}\\ &=U_{2}^{\dagger}U_{2}\\ &=\mathbb{I} \end{aligned}

Hence a product of unitary operator is unitary.

 ~\tag*{$\blacksquare$}

Exercise 1.6.4 It is assumed that you know (1) what a determinant is, (2) that detΩT=detΩ\det\Omega^\mathrm{T}=\det\Omega (T\mathrm{T} denotes transpose), (3) that the determinant of a product of matrices is the product of the determinants. [If you do not, verify these properties for a two-dimensional case

Ω=(αβγδ) \Omega=\begin{pmatrix} \alpha & \beta \\ \gamma & \delta \end{pmatrix}

with detΩ=(αδβγ)\det\Omega=(\alpha \delta-\beta \gamma).] Prove that the determinant of a unitary matrix is a complex number of unit modulus.

Solution. Suppose UU is the unitary matrix, which means that it satisfies

UU=I U^{\dagger}U=\mathbb{I}

Take determinant of the both sides, we get

det(UU)=det(I)det(U)det(U)=1det((UT))det(U)=1(det(UT))det(U)=1(det(U))det(U)=1det(U)2=1det(U)=1 \begin{aligned} \det(U^{\dagger}U)&=\det(\mathbb{I})\\ \det(U^{\dagger})\det(U)&=1\\ \det((U^{\mathrm{T}})^{\star})\det(U)&=1\\ (\det(U^{\mathrm{T}}))^{\star}\det(U)&=1\\ (\det(U))^{\star}\det(U)&=1\\ |\det(U)|^{2}&=1\\ |\det(U)|&=1 \end{aligned}

Therefore, det(U)\det (U) is a complex number of unit modulus.

 ~\tag*{$\blacksquare$}

Exercise 1.6.5 Verify that R(12πi)R\left(\frac{1}{2} \pi \mathbf{i}\right) is unitary (orthogonal) by examining its matrix.

Solution. We know from Example 1.6.1,

R(12πi)=(100001010) R\left(\frac{1}{2}\pi \mathbf{i}\right)=\begin{pmatrix} 1&0&0\\ 0&0&-1\\ 0&1&0 \end{pmatrix}

Therefore,

R(12πi)R(12πi)=(100001010)(100001010)=(100010001)=I R\left(\frac{1}{2}\pi \mathbf{i}\right)^{\dagger}R\left(\frac{1}{2}\pi \mathbf{i}\right)=\begin{pmatrix} 1&0&0\\ 0&0&1\\ 0&-1&0 \end{pmatrix}\begin{pmatrix} 1&0&0\\ 0&0&-1\\ 0&1&0 \end{pmatrix}=\begin{pmatrix} 1&0&0\\ 0&1&0\\ 0&0&1 \end{pmatrix}=\mathbb{I}

Hence, R(12πi)R\left(\frac{1}{2}\pi \mathbf{i}\right) is unitary.

 ~\tag*{$\blacksquare$}

Exercise 1.6.6 Verify that the following matrices are unitary:

121/2(1ii1),12(1+i1i1i1+i) \frac{1}{2^{1 / 2}}\begin{pmatrix} 1 & \mathrm{i} \\ \mathrm{i} & 1 \end{pmatrix}, \quad \frac{1}{2}\begin{pmatrix} 1+\mathrm{i} & 1-\mathrm{i} \\ 1-\mathrm{i} & 1+\mathrm{i} \end{pmatrix}

Verify that the determinant is of the form eiθe^{\mathrm{i} \theta} in each case. Are any of the above matrices Hermitian?

Solution.

  • 121/2(1ii1)\frac{1}{2^{1 / 2}}\begin{pmatrix} 1 & \mathrm{i} \\ \mathrm{i} & 1 \end{pmatrix} is unitary, since
121/2(1ii1)121/2(1ii1)=12(1ii1)(1ii1)=12(2002)=I \frac{1}{2^{1 / 2}}\begin{pmatrix} 1 & \mathrm{i} \\ \mathrm{i} & 1 \end{pmatrix}^{\dagger}\cdot \frac{1}{2^{1 / 2}}\begin{pmatrix} 1 & \mathrm{i} \\ \mathrm{i} & 1 \end{pmatrix}=\frac{1}{2}\begin{pmatrix} 1 & -\mathrm{i} \\ -\mathrm{i} & 1 \end{pmatrix}\begin{pmatrix} 1 & \mathrm{i} \\ \mathrm{i} & 1 \end{pmatrix}=\frac{1}{2}\begin{pmatrix} 2 & 0 \\ 0 & 2 \end{pmatrix}=\mathbb{I}

The determinant of 121/2(1ii1)\frac{1}{2^{1 / 2}}\begin{pmatrix} 1 & \mathrm{i} \\ \mathrm{i} & 1 \end{pmatrix} is of eiθ\mathrm{e}^{\mathrm{i}\theta} form, since

det[121/2(1ii1)]=(121/2)2(11ii)=1=eiθ, where θ=2kπ for kZ \det\left[\frac{1}{2^{1 / 2}}\begin{pmatrix} 1 & \mathrm{i} \\ \mathrm{i} & 1 \end{pmatrix}\right]=\left(\frac{1}{2^{1/2}}\right)^{2}(1\cdot 1-\mathrm{i}\cdot \mathrm{i})=1=\mathrm{e}^{\mathrm{i}\theta},~\text{where}~\theta=2k\pi~\text{for}~k\in\mathbb{Z}

121/2(1ii1)\frac{1}{2^{1 / 2}}\begin{pmatrix} 1 & \mathrm{i} \\ \mathrm{i} & 1 \end{pmatrix} is not Hermitian, since

121/2(1ii1)=121/2(1ii1)121/2(1ii1) \frac{1}{2^{1 / 2}}\begin{pmatrix} 1 & \mathrm{i} \\ \mathrm{i} & 1 \end{pmatrix}^{\dagger}=\frac{1}{2^{1 / 2}}\begin{pmatrix} 1 & -\mathrm{i} \\ -\mathrm{i} & 1 \end{pmatrix}\neq \frac{1}{2^{1 / 2}}\begin{pmatrix} 1 & \mathrm{i} \\ \mathrm{i} & 1 \end{pmatrix}
  • 12(1+i1i1i1+i)\frac{1}{2}\begin{pmatrix} 1+\mathrm{i} & 1-\mathrm{i} \\ 1-\mathrm{i} & 1+\mathrm{i} \end{pmatrix} is unitary, since
12(1+i1i1i1+i)12(1+i1i1i1+i)=14(1i1+i1+i1i)(1+i1i1i1+i)=14(4004)=I \begin{aligned} \frac{1}{2}\begin{pmatrix} 1+\mathrm{i} & 1-\mathrm{i} \\ 1-\mathrm{i} & 1+\mathrm{i} \end{pmatrix}^{\dagger}\cdot\frac{1}{2}\begin{pmatrix} 1+\mathrm{i} & 1-\mathrm{i} \\ 1-\mathrm{i} & 1+\mathrm{i} \end{pmatrix}&=\frac{1}{4}\begin{pmatrix} 1-\mathrm{i} & 1+\mathrm{i} \\ 1+\mathrm{i} & 1-\mathrm{i} \end{pmatrix}\begin{pmatrix} 1+\mathrm{i} & 1-\mathrm{i} \\ 1-\mathrm{i} & 1+\mathrm{i} \end{pmatrix}\\ &=\frac{1}{4}\begin{pmatrix} 4 & 0 \\ 0 & 4 \end{pmatrix}\\ &=\mathbb{I} \end{aligned}

The determinant of 12(1+i1i1i1+i)\frac{1}{2}\begin{pmatrix} 1+\mathrm{i} & 1-\mathrm{i} \\ 1-\mathrm{i} & 1+\mathrm{i} \end{pmatrix} is of eiθ\mathrm{e}^{\mathrm{i}\theta} form, since

det[12(1+i1i1i1+i)]=(12)2[(1+i)2(1i)2]=i \det\left[\frac{1}{2}\begin{pmatrix} 1+\mathrm{i} & 1-\mathrm{i} \\ 1-\mathrm{i} & 1+\mathrm{i} \end{pmatrix}\right]=\left(\frac{1}{2}\right)^{2}[(1+\mathrm{i})^{2}-(1-\mathrm{i})^{2}]=\mathrm{i}

where θ=2kπ+π2\theta=2k\pi+\frac{\pi}{2} for kZk\in\mathbb{Z}. 12(1+i1i1i1+i)\frac{1}{2}\begin{pmatrix} 1+\mathrm{i} & 1-\mathrm{i} \\ 1-\mathrm{i} & 1+\mathrm{i} \end{pmatrix} is not Hermitian, since

12(1+i1i1i1+i)=12(1i1+i1+i1i)12(1+i1i1i1+i) \frac{1}{2}\begin{pmatrix} 1+\mathrm{i} & 1-\mathrm{i} \\ 1-\mathrm{i} & 1+\mathrm{i} \end{pmatrix}^{\dagger}=\frac{1}{2}\begin{pmatrix} 1-\mathrm{i} & 1+\mathrm{i} \\ 1+\mathrm{i} & 1-\mathrm{i} \end{pmatrix}\neq \frac{1}{2}\begin{pmatrix} 1+\mathrm{i} & 1-\mathrm{i} \\ 1-\mathrm{i} & 1+\mathrm{i} \end{pmatrix}
 ~\tag*{$\blacksquare$}

1.7 Active and Passive Transformations

Exercise 1.7.1 The trace of a matrix is defined to be the sum of its diagonal matrix elements

TrΩ=iΩii \operatorname{Tr} \Omega=\sum_i \Omega_{i i}

Show that

(1) Tr(ΩΛ)=Tr(ΛΩ)\operatorname{Tr}(\Omega \Lambda)=\operatorname{Tr}(\Lambda \Omega).

(2) Tr(ΩΛθ)=Tr(ΛθΩ)=Tr(θΩΛ)\operatorname{Tr}(\Omega \Lambda \theta)=\operatorname{Tr}(\Lambda \theta \Omega)=\operatorname{Tr}(\theta \Omega \Lambda) (The permutations are \textit{cyclic}).

(3) The trace of an operator is unaffected by a unitary change of basis iUi|i\rangle \rightarrow U|i\rangle. [Equivalently, show TrΩ=Tr(UΩU)\operatorname{Tr} \Omega=\operatorname{Tr}\left(U^{\dagger} \Omega U\right).]

Solution. (1) Tr(ΩΛ)=i(ΩΛ)ii=ijΩijΛji=jiΛjiΩij=j(ΛΩ)jj=Tr(ΛΩ)\operatorname{Tr}(\Omega \Lambda)=\sum\limits_{i}(\Omega\Lambda)_{ii}=\sum\limits_{i}\sum\limits_{j}\Omega_{ij}\Lambda_{ji}=\sum\limits_{j}\sum\limits_{i}\Lambda_{ji}\Omega_{ij}=\sum\limits_{j}(\Lambda\Omega)_{jj}=\operatorname{Tr}(\Lambda\Omega).

(2)

Tr(ΩΛθ)=i(ΩΛθ)ii=ijkΩijΛjkθki=jkiΛjkθkiΩij=j(ΛθΩ)jj=Tr(ΛθΩ)=kijθkiΩijΛjk=k(θΩΛ)kk=Tr(θΩΛ) \begin{aligned} \operatorname{Tr}(\Omega\Lambda\theta)&=\sum_{i}(\Omega\Lambda\theta)_{ii}=\sum_{i}\sum_{j}\sum_{k}\Omega_{ij}\Lambda_{jk}\theta_{ki}\\ &=\sum_{j}\sum_{k}\sum_{i}\Lambda_{jk}\theta_{ki}\Omega_{ij}=\sum_{j}(\Lambda\theta\Omega)_{jj}=\operatorname{Tr}(\Lambda\theta\Omega)\\ &=\sum_{k}\sum_{i}\sum_{j}\theta_{ki}\Omega_{ij}\Lambda_{jk}=\sum_{k}(\theta\Omega\Lambda)_{kk}=\operatorname{Tr}(\theta\Omega\Lambda) \end{aligned}

(3) Tr(UΩU)=Tr(ΩUU)=Tr(ΩI)=Tr(Ω)\operatorname{Tr}(U^{\dagger}\Omega U)=\operatorname{Tr}(\Omega UU^{\dagger})=\operatorname{Tr}(\Omega\mathbb{I})=\operatorname{Tr}(\Omega).

 ~\tag*{$\blacksquare$}

Exercise 1.7.2 Show that the determinant of a matrix is unaffected by a unitary change of basis. [Equivalently show detΩ=det(UΩU)\det \Omega=\det\left(U^{\dagger} \Omega U\right).]

Solution.

det(UΩU)=detUdetΩdetU=detΩ(detUdetU)=detΩdet(UU)=detΩ1=detΩ. \begin{aligned} \det\left(U^{\dagger} \Omega U\right) & =\det U^{\dagger} \det \Omega \det U \\ & =\det\Omega\left(\det U^{\dagger} \det U\right) \\ & =\det \Omega \det\left(U^{\dagger} U\right) \\ & =\det \Omega \cdot 1 \\ & =\det \Omega . \end{aligned}
 ~\tag*{$\blacksquare$}

1.8 The Eigenvalue Problem

Exercise 1.8.1 (1) Find the eigenvalues and normalized eigenvectors of the matrix

Ω=(131020014) \Omega=\begin{pmatrix} 1&3&1\\ 0&2&0\\ 0&1&4 \end{pmatrix}

(2) Is the matrix Hermitian? Are the eigenvectors orthogonal?

Solution.

(1) To find the eigenvalues and normalized eigenvectors of the matrix Ω\Omega, we can compute the characteristic equation

det(ΩωI)=1ω3102ω0014ω=(1ω)(2ω)(4ω)=0 \det(\Omega-\omega\mathbb{I})=\left|\begin{matrix} 1-\omega & 3 & 1\\ 0 & 2-\omega & 0\\ 0 & 1 & 4-\omega \end{matrix}\right|=(1-\omega)(2-\omega)(4-\omega)=0

So the eigenvalues are

ω=1,2,4 \omega=1, 2, 4

The eigenvectors corresponding eigenvalues are

ω=1: (031010013)(x1x2x3)=0ω=1=(100)ω=2: (131000012)(x1x2x3)=0ω=2=130(521)ω=4: (331020010)(x1x2x3)=0ω=4=110(103) \begin{aligned} \omega=1: &~\begin{pmatrix} 0&3&1\\ 0&1&0\\ 0&1&3 \end{pmatrix}\begin{pmatrix} x_{1}\\ x_{2}\\ x_{3} \end{pmatrix}=0\quad &\Rightarrow\quad &|\omega=1\rangle=\begin{pmatrix} 1\\0\\0 \end{pmatrix}\\ \omega=2: &~\begin{pmatrix} -1&3&1\\ 0&0&0\\ 0&1&2 \end{pmatrix}\begin{pmatrix} x_{1}\\ x_{2}\\ x_{3} \end{pmatrix}=0\quad &\Rightarrow\quad &|\omega=2\rangle=\frac{1}{\sqrt{30}}\begin{pmatrix} 5\\2\\-1 \end{pmatrix}\\ \omega=4: &~\begin{pmatrix} -3&3&1\\ 0&-2&0\\ 0&1&0 \end{pmatrix}\begin{pmatrix} x_{1}\\ x_{2}\\ x_{3} \end{pmatrix}=0\quad &\Rightarrow\quad &|\omega=4\rangle=\frac{1}{\sqrt{10}}\begin{pmatrix} 1\\0\\3 \end{pmatrix} \end{aligned}

(2) Matrix Ω\Omega is not Hermitian, since

Ω=(100320104)Ω \Omega^{\dagger}=\begin{pmatrix} 1&0&0\\ 3&2&0\\ 1&0&4 \end{pmatrix}\neq\Omega

The eigenvectors are not orthogonal, since

ω=1ω=2=1×530+0×230+0×130=3060ω=1ω=4=1×110+0×0+0×310=10100ω=2ω=4=530×110+230×0+130×310=3150 \begin{aligned} &\langle\omega=1\mid\omega=2\rangle=1\times \frac{5}{\sqrt{30}}+0\times\frac{2}{\sqrt{30}}+0\times\frac{-1}{\sqrt{30}}=\frac{\sqrt{30}}{6}\neq 0\\ &\langle\omega=1\mid\omega=4\rangle=1\times\frac{1}{\sqrt{10}}+0\times 0+0\times\frac{3}{\sqrt{10}}=\frac{\sqrt{10}}{10}\neq 0\\&\langle\omega=2\mid\omega=4\rangle=\frac{5}{\sqrt{30}}\times\frac{1}{\sqrt{10}}+\frac{2}{\sqrt{30}}\times 0+\frac{-1}{\sqrt{30}}\times\frac{3}{\sqrt{10}}=\frac{\sqrt{3}}{15}\neq 0 \end{aligned}
 ~\tag*{$\blacksquare$}

Exercise 1.8.2 Consider the matrix

Ω=(001000100) \Omega=\begin{pmatrix} 0&0&1\\ 0&0&0\\ 1&0&0 \end{pmatrix}

(1) Is it Hermitian?

(2) Find its eigenvalues and eigenvectors.

(3) Verify that UΩUU^{\dagger} \Omega U is diagonal, UU being the matrix of eigenvectors of Ω\Omega.

Solution. (1) Matrix Ω\Omega is Hermitian since

Ω=Ω \Omega^{\dagger}=\Omega

(2) To find the eigenvalues and eigenvectors of the matrix Ω\Omega, we can compute the characteristic equation

det(ΩωI)=ω010ω010ω=ω3+ω=ω(ω+1)(ω1)=0 \det(\Omega-\omega\mathbb{I})=\left|\begin{matrix} -\omega & 0 & 1\\ 0 & -\omega & 0\\ 1 & 0 & -\omega \end{matrix}\right|=-\omega^{3}+\omega=-\omega(\omega+1)(\omega-1)=0

Therefore, eigenvalues are

ω=1,0,1 \omega=-1, 0, 1

The eigenvectors corresponding to eigenvalues are

ω=1: (101010101)(x1x2x3)=0ω=1=12(101)ω=0: (001000100)(x1x2x3)=0ω=0=(010)ω=1: (101010101)(x1x2x3)=0ω=1=12(101) \begin{aligned} \omega=-1: &~\begin{pmatrix} 1&0&1\\ 0&1&0\\ 1&0&1 \end{pmatrix}\begin{pmatrix} x_{1}\\ x_{2}\\ x_{3} \end{pmatrix}=0\quad &\Rightarrow\quad &|\omega=-1\rangle=\frac{1}{\sqrt{2}}\begin{pmatrix} 1\\0\\-1 \end{pmatrix}\\ \omega=0: &~\begin{pmatrix} 0&0&1\\ 0&0&0\\ 1&0&0 \end{pmatrix}\begin{pmatrix} x_{1}\\ x_{2}\\ x_{3} \end{pmatrix}=0\quad &\Rightarrow\quad &|\omega=0\rangle=\begin{pmatrix} 0\\1\\0 \end{pmatrix}\\ \omega=1: &~\begin{pmatrix} -1&0&1\\ 0&-1&0\\ 1&0&-1 \end{pmatrix}\begin{pmatrix} x_{1}\\ x_{2}\\ x_{3} \end{pmatrix}=0\quad &\Rightarrow\quad &|\omega=1\rangle=\frac{1}{\sqrt{2}}\begin{pmatrix} 1\\0\\1 \end{pmatrix} \end{aligned}

(3) If UU is the matrix of eigenvectors of Ω\Omega, then

U=(1201201012012)U=(1201201012012) U=\begin{pmatrix} \frac{1}{\sqrt{2}} & 0 & \frac{1}{\sqrt{2}}\\ 0 & 1 & 0\\ -\frac{1}{\sqrt{2}} & 0 & \frac{1}{\sqrt{2}} \end{pmatrix}\qquad U^{\dagger}=\begin{pmatrix} \frac{1}{\sqrt{2}} & 0 & -\frac{1}{\sqrt{2}}\\ 0 & 1 & 0\\ \frac{1}{\sqrt{2}} & 0 & \frac{1}{\sqrt{2}} \end{pmatrix}

We can compute

UΩU=(1201201012012)(001000100)(1201201012012)=(1201200012012)(1201201012012)=(100000001) \begin{aligned} U^{\dagger}\Omega U&=\begin{pmatrix} \frac{1}{\sqrt{2}} & 0 & -\frac{1}{\sqrt{2}}\\ 0 & 1 & 0\\ \frac{1}{\sqrt{2}} & 0 & \frac{1}{\sqrt{2}} \end{pmatrix}\begin{pmatrix} 0&0&1\\ 0&0&0\\ 1&0&0 \end{pmatrix}\begin{pmatrix} \frac{1}{\sqrt{2}} & 0 & \frac{1}{\sqrt{2}}\\ 0 & 1 & 0\\ -\frac{1}{\sqrt{2}} & 0 & \frac{1}{\sqrt{2}} \end{pmatrix}\\ &=\begin{pmatrix} -\frac{1}{\sqrt{2}} & 0 & \frac{1}{\sqrt{2}}\\ 0 & 0 & 0\\ \frac{1}{\sqrt{2}} & 0 & \frac{1}{\sqrt{2}} \end{pmatrix}\begin{pmatrix} \frac{1}{\sqrt{2}} & 0 & \frac{1}{\sqrt{2}}\\ 0 & 1 & 0\\ -\frac{1}{\sqrt{2}} & 0 & \frac{1}{\sqrt{2}} \end{pmatrix}\\ &=\begin{pmatrix} -1 & 0 & 0\\ 0 & 0 & 0\\ 0 & 0 & 1 \end{pmatrix} \end{aligned}

This is a diagonal matrix.

 ~\tag*{$\blacksquare$}

Exercise 1.8.3 Consider the Hermitian matrix

Ω=12(200031013) \Omega=\frac{1}{2}\begin{pmatrix} 2 & 0 & 0\\ 0 & 3 & -1\\ 0 & -1 & 3 \end{pmatrix}

(1) Show that ω1=ω2=1\omega_{1}=\omega_{2}=1; ω3=2\omega_{3}=2.

(2) Show that ω=2|\omega=2\rangle is any vector of the form

1(2a2)1/2(0aa) \frac{1}{(2a^{2})^{1/2}}\begin{pmatrix} 0\\a\\-a \end{pmatrix}

(3) Show that the ω=1\omega=1 eigenspace contains all vectors of the form

1(b2+2c2)1/2(bcc) \frac{1}{(b^{2}+2c^{2})^{1/2}}\begin{pmatrix} b\\c\\c \end{pmatrix}

either by feeding ω=1\omega=1 into the equations or by requiring that the ω=1\omega=1 eigenspace be orthogonal to ω=2|\omega=2\rangle.

Solution. (1) The characteristic equation is

det(ΩωI)=1ω00032ω1201232ω=(1ω)(32ω)2(1ω)(12)2=(1ω)[(32ω)214]=(1ω)(ω23ω+2)=(1ω)(ω1)(ω2)=0 \begin{aligned} \det(\Omega-\omega\mathbb{I})&=\left|\begin{matrix} 1-\omega & 0 & 0\\ 0 & \frac{3}{2}-\omega & -\frac{1}{2}\\ 0 & -\frac{1}{2} & \frac{3}{2}-\omega \end{matrix}\right|\\ &=(1-\omega)\left(\frac{3}{2}-\omega\right)^{2}-(1-\omega)\left(-\frac{1}{2}\right)^{2}\\ &=(1-\omega)\left[\left(\frac{3}{2}-\omega\right)^{2}-\frac{1}{4}\right]=(1-\omega)(\omega^{2}-3\omega+2)\\ &=(1-\omega)(\omega-1)(\omega-2)=0 \end{aligned}

Then the eigenvalues are

ω1=ω2=1ω3=2 \omega_{1}=\omega_{2}=1\quad\omega_{3}=2

(2) To get the eigenvector corresponding to eigenvalue ω=2\omega=2, we need to solve the equation

(1000121201212)(x1x2x3)=0{x1=0x2+x3=0 \begin{pmatrix} -1 & 0 & 0\\ 0 & -\frac{1}{2} & -\frac{1}{2}\\ 0 & -\frac{1}{2} & -\frac{1}{2} \end{pmatrix}\begin{pmatrix} x_{1}\\x_{2}\\x_{3} \end{pmatrix}=0\quad \Rightarrow \quad \left\{ \begin{aligned} x_{1}&=0\\ x_{2}+x_{3}&=0 \end{aligned} \right.

Set x2=ax_{2}=a, we have x3=ax_{3}=-a. Therefore,

ω=2=12a2(0aa) |\omega=2\rangle=\frac{1}{\sqrt{2a^{2}}}\begin{pmatrix} 0\\a\\-a \end{pmatrix}

(3) For ω=1\omega=1:

(0000121201212)(x1x2x3)=0 \begin{pmatrix} 0 & 0 & 0\\ 0 & \frac{1}{2} & -\frac{1}{2}\\ 0 & -\frac{1}{2} & \frac{1}{2} \end{pmatrix} \begin{pmatrix} x_{1}\\x_{2}\\x_{3} \end{pmatrix}=0

x1x_{1} is arbitrary, set x1=bx_{1}=b. x2x3=0x_{2}-x_{3}=0, set x2=cx_{2}=c, then x3=cx_{3}=c. Therefore, the eigenvector corresponding ω=1\omega=1 is of the form

ω=1=1b2+2c2(bcc). |\omega=1\rangle=\frac{1}{\sqrt{b^{2}+2c^{2}}}\begin{pmatrix} b\\c\\c \end{pmatrix}.
 ~\tag*{$\blacksquare$}

Exercise 1.8.4 An arbitrary n×nn \times n matrix need not have nn eigenvectors. Consider as an example

Ω=(4112) \Omega=\begin{pmatrix} 4 & 1 \\ -1 & 2 \end{pmatrix}

(1) Show that ω1=ω2=3\omega_1=\omega_2=3.

(2) By feeding in this value show we get only one eigenvector of the form

1(2a2)1/2(+aa) \frac{1}{\left(2 a^2\right)^{1 / 2}}\begin{pmatrix} +a \\ -a \end{pmatrix}

We cannot find another one that is linear independent.

Solution. (1) The characteristic equation is

det(ΩωI)=4ω112ω=(4ω)(2ω)+1=ω26ω+9=0 \det(\Omega-\omega\mathbb{I})=\left|\begin{matrix} 4-\omega & 1\\ -1 & 2-\omega \end{matrix}\right|=(4-\omega)(2-\omega)+1=\omega^{2}-6\omega+9=0

Thus the eigenvalues are

ω1=ω2=3 \omega_{1}=\omega_{2}=3

(2) By feeding this eigenvalue ω=3\omega=3, we get the equation

(1111)(x1x2)=0x1+x2=0 \begin{pmatrix} 1 & 1\\ -1 & -1 \end{pmatrix} \begin{pmatrix} x_{1}\\x_{2} \end{pmatrix}=0\qquad \Rightarrow \qquad x_{1}+x_{2}=0

Set x1=ax_{1}=a, we have x2=ax_{2}=-a. Therefore, the eigenvector is of the form

ω=3=12a2(aa). |\omega=3\rangle=\frac{1}{\sqrt{2a^{2}}}\begin{pmatrix} a\\-a \end{pmatrix}.

This is the only eigenvector we can find.

 ~\tag*{$\blacksquare$}

Exercise 1.8.5 Consider the matrix

Ω=(cosθsinθsinθcosθ) \Omega=\begin{pmatrix} \cos \theta & \sin \theta \\ -\sin \theta & \cos \theta \end{pmatrix}

(1) Show that it is unitary.

(2) Show that its eigenvalues are eiθ\mathrm{e}^{\mathrm{i} \theta} and eiθ\mathrm{e}^{-\mathrm{i} \theta}.

(3) Find the corresponding eigenvectors; show that they are orthogonal.

(4) Verify that UΩU=U^{\dagger} \Omega U= (diagonal matrix), where UU is the matrix of eigenvectors of Ω\Omega.

Solution. (1) Matrix Ω\Omega is unitary, since

ΩΩ=(cosθsinθsinθcosθ)(cosθsinθsinθcosθ)=(cos2θ+sin2θcosθsinθsinθcosθsinθcosθcosθsinθsin2θ+cos2θ)=(1001)=I \begin{aligned} \Omega^{\dagger}\Omega&=\begin{pmatrix} \cos \theta & -\sin \theta \\ \sin \theta & \cos \theta \end{pmatrix}\begin{pmatrix} \cos \theta & \sin \theta \\ -\sin \theta & \cos \theta \end{pmatrix}\\ &=\begin{pmatrix} \cos^{2}\theta+\sin^{2}\theta & \cos\theta\sin\theta-\sin\theta\cos\theta\\ \sin\theta\cos\theta-\cos\theta\sin\theta & \sin^{2}\theta+\cos^{2}\theta \end{pmatrix}\\ &=\begin{pmatrix} 1 & 0\\ 0 & 1 \end{pmatrix}\\ &=\mathbb{I} \end{aligned}

(2) Solve the characteristic equation

det(ΩωI)=cosθωsinθsinθcosθω=ω22ωcosθ+1=0 \det(\Omega-\omega\mathbb{I})=\left|\begin{matrix} \cos\theta-\omega & \sin\theta\\ -\sin\theta & \cos\theta-\omega \end{matrix}\right|=\omega^{2}-2\omega\cos\theta+1=0

By Euler's formula, we get the eigenvalues

ω=cosθ±isinθ=e±iθ\omega=\cos\theta\pm \mathrm{i}\sin\theta=\mathrm{e}^{\pm\mathrm{i}\theta}

(3) By feeding this eigenvalue, we get the equations

ω=eiθ: (isinθsinθsinθisinθ)(x1x2)=0ix1+x2=0ω=eiθ: (isinθsinθsinθisinθ)(x1x2)=0ix1+x2=0 \begin{aligned} \omega&=\mathrm{e}^{-\mathrm{i}\theta}: &~\begin{pmatrix} \mathrm{i}\sin\theta & \sin\theta\\ -\sin\theta &\mathrm{i} \sin\theta \end{pmatrix}\begin{pmatrix} x_{1}\\x_{2} \end{pmatrix}=0\quad &\Rightarrow\quad \mathrm{i}x_{1}+x_{2}=0\\ \omega&=\mathrm{e}^{\mathrm{i}\theta}: &~\begin{pmatrix} -\mathrm{i}\sin\theta & \sin\theta\\ -\sin\theta &-\mathrm{i} \sin\theta \end{pmatrix}\begin{pmatrix} x_{1}\\x_{2} \end{pmatrix}=0\quad &\Rightarrow\quad -\mathrm{i}x_{1}+x_{2}=0 \end{aligned}

Thus the corresponding eigenvectors are

ω=eiθ=12(1i)ω=eiθ=12(1i) \begin{aligned} &|\omega=\mathrm{e}^{-\mathrm{i}\theta}\rangle=\frac{1}{\sqrt{2}}\begin{pmatrix} 1\\-\mathrm{i} \end{pmatrix}\\ &|\omega=\mathrm{e}^{\mathrm{i}\theta}\rangle=\frac{1}{\sqrt{2}}\begin{pmatrix} 1\\\mathrm{i} \end{pmatrix} \end{aligned}

They are orthogonal since

ω=eiθω=eiθ=12(1i)(1i)=12(1+i2)=0 \langle \omega=\mathrm{e}^{-\mathrm{i}\theta}\mid \omega=\mathrm{e}^{\mathrm{i}\theta}\rangle=\frac{1}{2}(1\quad\mathrm{i})\begin{pmatrix} 1\\\mathrm{i} \end{pmatrix}=\frac{1}{2}(1+\mathrm{i}^{2})=0

(4) The matrix of eigenvectors of Ω\Omega is

U=(1212i2i2)U=\begin{pmatrix} \frac{1}{\sqrt{2}} & \frac{1}{\sqrt{2}}\\ -\frac{\mathrm{i}}{\sqrt{2}} & \frac{\mathrm{i}}{\sqrt{2}} \end{pmatrix}

Then

UΩU=(12i212i2)(cosθsinθsinθcosθ)(1212i2i2)=(12(cosθisinθ)12(sinθ+icosθ)12(cosθ+isinθ)12(sinθicosθ))(1212i2i2)=(12eiθi2eiθ12eiθi2eiθ)(1212i2i2)=(eiθ00eiθ) \begin{aligned} U^{\dagger}\Omega U&=\begin{pmatrix} \frac{1}{\sqrt{2}} & \frac{\mathrm{i}}{\sqrt{2}}\\ \frac{1}{\sqrt{2}} & -\frac{\mathrm{i}}{\sqrt{2}} \end{pmatrix}\begin{pmatrix} \cos \theta & \sin \theta \\ -\sin \theta & \cos \theta \end{pmatrix}\begin{pmatrix} \frac{1}{\sqrt{2}} & \frac{1}{\sqrt{2}}\\ -\frac{\mathrm{i}}{\sqrt{2}} & \frac{\mathrm{i}}{\sqrt{2}} \end{pmatrix}\\ &=\begin{pmatrix} \frac{1}{\sqrt{2}}(\cos\theta-\mathrm{i}\sin\theta) & \frac{1}{\sqrt{2}}(\sin\theta+\mathrm{i}\cos\theta)\\ \frac{1}{\sqrt{2}}(\cos\theta+\mathrm{i}\sin\theta) & \frac{1}{\sqrt{2}}(\sin\theta-\mathrm{i}\cos\theta) \end{pmatrix}\begin{pmatrix} \frac{1}{\sqrt{2}} & \frac{1}{\sqrt{2}}\\ -\frac{\mathrm{i}}{\sqrt{2}} & \frac{\mathrm{i}}{\sqrt{2}} \end{pmatrix}\\ &=\begin{pmatrix} \frac{1}{\sqrt{2}}\mathrm{e}^{-\mathrm{i}\theta} & \frac{\mathrm{i}}{\sqrt{2}}\mathrm{e}^{-\mathrm{i}\theta}\\ \frac{1}{\sqrt{2}}\mathrm{e}^{\mathrm{i}\theta} & -\frac{\mathrm{i}}{\sqrt{2}}\mathrm{e}^{\mathrm{i}\theta} \end{pmatrix}\begin{pmatrix} \frac{1}{\sqrt{2}} & \frac{1}{\sqrt{2}}\\ -\frac{\mathrm{i}}{\sqrt{2}} & \frac{\mathrm{i}}{\sqrt{2}} \end{pmatrix}\\ &=\begin{pmatrix} \mathrm{e}^{-\mathrm{i}\theta} & 0\\ 0 & \mathrm{e}^{\mathrm{i}\theta} \end{pmatrix} \end{aligned}

is diagonal.

 ~\tag*{$\blacksquare$}

Exercise 1.8.6 (1) We have seen that the determinant of a matrix is unchanged under a unitary change of basis. Argue now that

detΩ= product of eigenvalues of Ω=i=1nωi \operatorname{det} \Omega=\text { product of eigenvalues of } \Omega=\prod_{i=1}^n \omega_i

for a Hermitian or unitary Ω\Omega.

(2) Using the invariance of the trace under the same transformation, show that

TrΩ=i=1nωi \operatorname{Tr} \Omega=\sum_{i=1}^n \omega_i

Solution. (1) Suppose UU is the unitary matrix that transforms Ω\Omega into a diagonal matrix DD with Ω\Omega's eigenvalues ωi\omega_{i} on its diagonal. Then

detΩ=det(UΩU)=detD=i=1nωi \det\Omega=\det(U^{\dagger}\Omega U)=\det D=\prod_{i=1}^{n}\omega_{i}

(2) By using the same transformation, we have

TrΩ=Tr(UΩU)=TrD=i=1nωi \operatorname{Tr}\Omega=\operatorname{Tr}(U^{\dagger}\Omega U)=\operatorname{Tr} D=\sum_{i=1}^{n}\omega_{i}
 ~\tag*{$\blacksquare$}

Exercise 1.8.7 By using the results on the trace and determinant from the last problem, show that the eigenvalues of the matrix

Ω=(1221) \Omega=\begin{pmatrix} 1 & 2 \\ 2 & 1 \end{pmatrix}

are 33 and 1-1. Verify this by explicit computation. Note that the Hermitian nature of the matrix is an essential ingredient.

Solution. According to Exercise 1.8.6, we have

{ω1×ω2=detΩ=1×12×2=3ω1+ω2=1+1=2 \left\{ \begin{aligned} \omega_{1}\times \omega_{2}&=\det\Omega=1\times 1-2\times 2=-3\\ \omega_{1}+\omega_{2}&=1+1=2 \end{aligned}\right.

Solving the equation, we get

{ω1=1ω2=3 \left\{\begin{aligned} \omega_{1}&=-1\\ \omega_{2}&=3 \end{aligned}\right.

For verification, we can calculate the characteristic equation

det(ΩωI)=1ω221ω=(1ω)24=(1ω+2)(1ω2)=0 \det(\Omega-\omega\mathbb{I})=\left|\begin{matrix} 1-\omega & 2\\ 2 & 1-\omega \end{matrix}\right|=(1-\omega)^{2}-4=(1-\omega+2)(1-\omega-2)=0

We can get the eigenvalues

{ω1=1ω2=3 \left\{\begin{aligned} \omega_{1}&=-1\\ \omega_{2}&=3 \end{aligned}\right.
 ~\tag*{$\blacksquare$}

Exercise 1.8.8 Consider Hermitian matrices M1,M2,M3,M4M^1, M^2, M^3, M^4 that obey

MiMj+MjMi=2δijI,i,j=1,,4 M^i M^j+M^j M^i=2 \delta^{i j} \mathbb{I}, \quad i, j=1, \ldots, 4

(1) Show that the eigenvalues of MiM^i are ±1\pm 1. (Hint: go to the eigenbasis of MiM^i, and use the equation for i=ji=j.)

(2) By considering the relation

MiMi=MjMi for ij M^i M^i=-M^j M^i \quad \text { for } i \neq j

show that MiM^i are traceless. [Hint: Tr(ACB)=Tr(CBA)\operatorname{Tr}(ACB)=\operatorname{Tr}(CBA).]

(3) Show that they cannot be odd-dimensional matrices.

Solution. (1) Start with equation

MiMj+MjMi=2δijI M^{i}M^{j}+M^{j}M^{i}=2\delta^{ij}\mathbb{I}

Take i=ji=j, we get

MiMi=I M^{i}M^{i}=\mathbb{I}

Apply MiMiM^{i}M^{i} to eigenvector ω|\omega\rangle of MiM^{i}, we have

MiMiω=Mi(ωω)=ω2ωMiMiω=Iω=ω \begin{aligned} M^{i}M^{i}|\omega\rangle&=M^{i}(\omega|\omega\rangle)=\omega^{2}|\omega\rangle\\ M^{i}M^{i}|\omega\rangle&=\mathbb{I}|\omega\rangle=|\omega\rangle \end{aligned}

Therefore,

ω2=1ω=±1 \begin{aligned} \omega^{2}&=1\\ \omega&=\pm 1 \end{aligned}

(2) From the relation

MiMj=MjMiMjMiMj=MjMjMi=Mi \begin{aligned} M^{i}M^{j}&=-M^{j}M^{i}\\ M^{j}M^{i}M^{j}&=-M^{j}M^{j}M^{i}=-M^{i} \end{aligned}

We can take the trace of MiM^{i} to get

TrMi=Tr(MjMiMj)=Tr(MjMiMj)=Tr(MiMjMj)=Tr(MiI)=Tr(Mi)=0 \begin{aligned} \operatorname{Tr} M^{i}&=\operatorname{Tr}(-M^{j}M^{i}M^{j})\\ &=-\operatorname{Tr}(M^{j}M^{i}M^{j})\\ &=-\operatorname{Tr}(M^{i}M^{j}M^{j})\\ &=-\operatorname{Tr}(M^{i}\mathbb{I})\\ &=-\operatorname{Tr}(M^{i})\\ &=0 \end{aligned}

MiM^{i} is traceless.

(3) According to 1.8.6,

TrMi=k=1nωk \operatorname{Tr} M^{i}=\sum_{k=1}^{n}\omega_{k}

where nn is the dimension of the matrix. Since ωk=±1\omega_{k}=\pm 1, TrMi\operatorname{Tr} M^{i} can be zero only if nn is even. (The sum of an odd number of odd numbers is still odd, and cannot be zero.)

 ~\tag*{$\blacksquare$}

Exercise 1.8.9 A collection of masses mαm_\alpha, located at rα\mathbf{r}_\alpha and rotating with angular velocity ω\boldsymbol{\omega} around a common axis has an angular momentum

l=αmα(rα×vα) \mathbf{l}=\sum_\alpha m_\alpha\left(\mathbf{r}_\alpha \times \mathbf{v}_\alpha\right)

where vα=ω×rα\mathbf{v}_\alpha=\boldsymbol{\omega} \times \mathbf{r}_\alpha is the velocity of mαm_\alpha. By using the identity

A×(B×C)=B(AC)C(AB) \mathbf{A} \times(\mathbf{B} \times \mathbf{C})=\mathbf{B}(\mathbf{A} \cdot \mathbf{C})-\mathbf{C}(\mathbf{A} \cdot \mathbf{B})

show that each Cartesian component lil_i of l\mathbf{l} is given by

li=jMijωj l_i=\sum_j M_{i j} \omega_j

where

Mij=αmα[rα2δij(rα)i(rα)j] M_{i j}=\sum_\alpha m_\alpha\left[r_\alpha^2 \delta_{i j}-\left(\mathbf{r}_\alpha\right)_i\left(\mathbf{r}_\alpha\right)_j\right]

or in Dirac notation

l=Mω |l\rangle=M|\omega\rangle

(1) Will the angular momentum and angular velocity always be parallel?

(2) Show that the moment of inertia matrix MijM_{i j} is Hermitian.

(3) Argue now that there exist three directions for ω\boldsymbol{\omega} such that l\mathbf{l} and ω\boldsymbol{\omega} will be parallel. How are these directions to be found?

(4) Consider the moment of inertia matrix of a sphere. Due to the complete symmetry of the sphere, it is clear that every direction is its eigendirection for rotation. What does this say about the three eigenvalues of the matrix MM?

Solution. Start from the angular momentum

l=αmαrα×(ω×rα)=αmα[ω(rαrα)rα(rαω)]=αmα[ωrα2rα(rαω)]\begin{aligned} \mathbf{l}&=\sum_{\alpha} m_{\alpha} \mathbf{r}_{\alpha}\times(\boldsymbol{\omega}\times \mathbf{r}_{\alpha})\\ &=\sum_{\alpha} m_{\alpha} [\boldsymbol{\omega}(\mathbf{r}_{\alpha}\cdot \mathbf{r}_{\alpha})-\mathbf{r}_{\alpha}(\mathbf{r}_{\alpha}\cdot\boldsymbol{\omega})]\\ &=\sum_{\alpha} m_{\alpha} [\boldsymbol{\omega}\,r_{\alpha}^{2}-\mathbf{r}_{\alpha}(\mathbf{r}_{\alpha}\cdot\boldsymbol{\omega})] \end{aligned}

Writing in components, we get

li=αmα[ωirα2(rα)i(rαω)]=αmα[ωirα2(rα)ij(rα)jωj]=αmα[jδijωjrα2(rα)ij(rα)jωj]=jαmα[rα2δij(rα)i(rα)j]ωjjMijωj\begin{aligned} l_{i}&=\sum_{\alpha} m_{\alpha}[\omega_{i}\,r_{\alpha}^{2}-(\mathbf{r}_{\alpha})_{i}\,(\mathbf{r}_{\alpha}\cdot\boldsymbol{\omega})]\\ &=\sum_{\alpha} m_{\alpha}[\omega_{i}\,r_{\alpha}^{2}-(\mathbf{r}_{\alpha})_{i}\,\sum_{j}(\mathbf{r}_{\alpha})_{j}\omega_{j}]\\ &=\sum_{\alpha} m_{\alpha}[\sum_{j}\delta_{ij}\omega_{j}\,r_{\alpha}^{2}-(\mathbf{r}_{\alpha})_{i}\,\sum_{j}(\mathbf{r}_{\alpha})_{j}\omega_{j}]\\ &=\sum_{j}\sum_{\alpha} m_{\alpha}[r_{\alpha}^{2}\,\delta_{ij}-(\mathbf{r}_{\alpha})_{i}\,(\mathbf{r}_{\alpha})_{j}]\omega_{j}\\ &\equiv \sum_{j}M_{ij}\omega_{j} \end{aligned}

where Mijαmα[rα2δij(rα)i(rα)j]M_{ij}\equiv\sum_{\alpha} m_{\alpha}[r_{\alpha}^{2}\,\delta_{ij}-(\mathbf{r}_{\alpha})_{i}\,(\mathbf{r}_{\alpha})_{j}]. Or in Dirac notation,

l=Mω|l\rangle=M|\omega\rangle

(1) No. The angular momentum and angular velocity are not parallel unless ω|\omega\rangle is an eigenvector of $M

(2) Mji=(αmα[rα2δji(rα)j(rα)i])=αmα[rα2δij(rα)i(rα)j]=MijM_{ji}^{\star}=(\sum_{\alpha} m_{\alpha}[r_{\alpha}^{2}\,\delta_{ji}-(\mathbf{r}_{\alpha})_{j}\,(\mathbf{r}_{\alpha})_{i}])^{\star}=\sum_{\alpha} m_{\alpha}[r_{\alpha}^{2}\,\delta_{ij}-(\mathbf{r}_{\alpha})_{i}\,(\mathbf{r}_{\alpha})_{j}]=M_{ij}.

(3) Since MM is Hermitian, we can always find three eigenvectors which are orthogonal to each other by solving the eigen-problem Mω=ωωM|\omega\rangle=\omega|\omega\rangle. And these three eigenvectors denote the three directions for ω\boldsymbol{\omega} we want to find in the 3-dimensional Euclidean space.

(4) The complete symmetry of sphere means all directions are equivalent eigendirections. Therefore, the eigenvalues are degenerate.

 ~\tag*{$\blacksquare$}

Exercise 1.8.10 By considering the commutator, show that the following Hermitian matrices may be simultaneously diagonalized. Find the eigenvectors common to both and verify that under a unitary transformation to this basis, both matrices are diagonalized.

Ω=(101000101),Λ=(211101112)\Omega=\begin{pmatrix} 1 & 0 & 1 \\ 0 & 0 & 0 \\ 1 & 0 & 1 \end{pmatrix}, \quad \Lambda=\begin{pmatrix} 2 & 1 & 1 \\ 1 & 0 & -1 \\ 1 & -1 & 2 \end{pmatrix}

Since Ω\Omega is degenerate and Λ\Lambda is not, you must be prudent in deciding which matrix dictates the choice of basis.

Solution. Since the two Hermitian matrices commute

[Ω,Λ]=ΩΛΛΩ=(101000101)(211101112)(211101112)(101000101)=(303000303)(303000303)=0\begin{aligned} \left[\Omega,\Lambda\right]&=\Omega\Lambda-\Lambda\Omega\\ &=\begin{pmatrix} 1 & 0 & 1 \\ 0 & 0 & 0 \\ 1 & 0 & 1 \end{pmatrix}\begin{pmatrix} 2 & 1 & 1 \\ 1 & 0 & -1 \\ 1 & -1 & 2 \end{pmatrix}-\begin{pmatrix} 2 & 1 & 1 \\ 1 & 0 & -1 \\ 1 & -1 & 2 \end{pmatrix}\begin{pmatrix} 1 & 0 & 1 \\ 0 & 0 & 0 \\ 1 & 0 & 1 \end{pmatrix}\\ &=\begin{pmatrix} 3 & 0 & 3 \\ 0 & 0 & 0 \\ 3 & 0 & 3 \end{pmatrix}-\begin{pmatrix} 3 & 0 & 3 \\ 0 & 0 & 0 \\ 3 & 0 & 3 \end{pmatrix}\\ &=0 \end{aligned}

They can be diagonalized simultaneously. We choose Λ\Lambda's characteristic equation

det(ΛλI)=2λ111λ1112λ=(λ+1)(2λ)(λ3)=0\det(\Lambda-\lambda\mathbb{I})=\left| \begin{matrix} 2-\lambda & 1 & 1\\ 1 & -\lambda & -1\\ 1 & -1 & 2-\lambda \end{matrix} \right|=(\lambda+1)(2-\lambda)(\lambda-3)=0

The eigenvalues are

λ=1,2,3\lambda=-1, 2, 3

Then the eigenvectors corresponding the eigenvalues are

λ=1: (311111113)(x1x2x3)=0λ=1=16(121)λ=2: (011121110)(x1x2x3)=0λ=2=13(111)λ=3: (111131111)(x1x2x3)=0λ=3=12(101)\begin{aligned} &\lambda=-1: &~\begin{pmatrix} 3&1&1\\ 1&1&-1\\ 1&-1&3 \end{pmatrix}\begin{pmatrix} x_{1}\\x_{2}\\x_{3} \end{pmatrix}=0\quad &\Rightarrow \quad |\lambda=-1\rangle=\frac{1}{\sqrt{6}}\begin{pmatrix} 1\\-2\\-1 \end{pmatrix}\\ &\lambda=2: &~\begin{pmatrix} 0&1&1\\ 1&-2&-1\\ 1&-1&0 \end{pmatrix}\begin{pmatrix} x_{1}\\x_{2}\\x_{3} \end{pmatrix}=0\quad &\Rightarrow \quad |\lambda=2\rangle=\frac{1}{\sqrt{3}}\begin{pmatrix} 1\\1\\-1 \end{pmatrix}\\ &\lambda=3: &~\begin{pmatrix} -1&1&1\\ 1&-3&-1\\ 1&-1&-1 \end{pmatrix}\begin{pmatrix} x_{1}\\x_{2}\\x_{3} \end{pmatrix}=0\quad &\Rightarrow \quad |\lambda=3\rangle=\frac{1}{\sqrt{2}}\begin{pmatrix} 1\\0\\1 \end{pmatrix} \end{aligned}

Then the matrix of eigenvectors of Λ\Lambda is

U=(16131226130161312)U=\begin{pmatrix} \frac{1}{\sqrt{6}}&\frac{1}{\sqrt{3}}&\frac{1}{\sqrt{2}}\\ -\frac{2}{\sqrt{6}}&\frac{1}{\sqrt{3}}&0\\ -\frac{1}{\sqrt{6}}&-\frac{1}{\sqrt{3}}&\frac{1}{\sqrt{2}} \end{pmatrix}

To verify Ω\Omega and Λ\Lambda are simultanelously diagonalized:

UΩU=(16261613131312012)(101000101)(16131226130161312)=(000000202)(16131226130161312)=(000000002)\begin{aligned} U^{\dagger}\Omega U&=\begin{pmatrix} \frac{1}{\sqrt{6}}&-\frac{2}{\sqrt{6}}&-\frac{1}{\sqrt{6}}\\ \frac{1}{\sqrt{3}}&\frac{1}{\sqrt{3}}&-\frac{1}{\sqrt{3}}\\ \frac{1}{\sqrt{2}}&0&\frac{1}{\sqrt{2}} \end{pmatrix}\begin{pmatrix} 1 & 0 & 1 \\ 0 & 0 & 0 \\ 1 & 0 & 1 \end{pmatrix}\begin{pmatrix} \frac{1}{\sqrt{6}}&\frac{1}{\sqrt{3}}&\frac{1}{\sqrt{2}}\\ -\frac{2}{\sqrt{6}}&\frac{1}{\sqrt{3}}&0\\ -\frac{1}{\sqrt{6}}&-\frac{1}{\sqrt{3}}&\frac{1}{\sqrt{2}} \end{pmatrix}\\ &=\begin{pmatrix} 0&0&0\\ 0&0&0\\ \sqrt{2}&0&\sqrt{2} \end{pmatrix}\begin{pmatrix} \frac{1}{\sqrt{6}}&\frac{1}{\sqrt{3}}&\frac{1}{\sqrt{2}}\\ -\frac{2}{\sqrt{6}}&\frac{1}{\sqrt{3}}&0\\ -\frac{1}{\sqrt{6}}&-\frac{1}{\sqrt{3}}&\frac{1}{\sqrt{2}} \end{pmatrix}\\ &=\begin{pmatrix} 0 & 0 & 0 \\ 0 & 0 & 0 \\ 0 & 0 & 2 \end{pmatrix} \end{aligned}
UΛU=(16261613131312012)(211101112)(16131226130161312)=(16261623232332032)(16131226130161312)=(100020003)\begin{aligned} U^{\dagger}\Lambda U&=\begin{pmatrix} \frac{1}{\sqrt{6}}&-\frac{2}{\sqrt{6}}&-\frac{1}{\sqrt{6}}\\ \frac{1}{\sqrt{3}}&\frac{1}{\sqrt{3}}&-\frac{1}{\sqrt{3}}\\ \frac{1}{\sqrt{2}}&0&\frac{1}{\sqrt{2}} \end{pmatrix}\begin{pmatrix} 2 & 1 & 1 \\ 1 & 0 & -1 \\ 1 & -1 & 2 \end{pmatrix}\begin{pmatrix} \frac{1}{\sqrt{6}}&\frac{1}{\sqrt{3}}&\frac{1}{\sqrt{2}}\\ -\frac{2}{\sqrt{6}}&\frac{1}{\sqrt{3}}&0\\ -\frac{1}{\sqrt{6}}&-\frac{1}{\sqrt{3}}&\frac{1}{\sqrt{2}} \end{pmatrix}\\ &=\begin{pmatrix} -\frac{1}{\sqrt{6}}&\frac{2}{\sqrt{6}}&\frac{1}{\sqrt{6}}\\ \frac{2}{\sqrt{3}}&\frac{2}{\sqrt{3}}&-\frac{2}{\sqrt{3}}\\ \frac{3}{\sqrt{2}}&0&\frac{3}{\sqrt{2}} \end{pmatrix}\begin{pmatrix} \frac{1}{\sqrt{6}}&\frac{1}{\sqrt{3}}&\frac{1}{\sqrt{2}}\\ -\frac{2}{\sqrt{6}}&\frac{1}{\sqrt{3}}&0\\ -\frac{1}{\sqrt{6}}&-\frac{1}{\sqrt{3}}&\frac{1}{\sqrt{2}} \end{pmatrix}\\ &=\begin{pmatrix} -1 & 0 & 0 \\ 0 & 2 & 0 \\ 0 & 0 & 3 \end{pmatrix} \end{aligned}
 ~\tag*{$\blacksquare$}

Exercise 1.8.11 Consider the coupled mass problem discussed above.

(1) Given that the initial state is 1|1\rangle, in which the first mass is displaced by unity and the second is left alone, calculate 1(t)|1(t)\rangle by following the algorithm.

(2) Compare your result with that following from Eq. (1.8.39).

Solution.

(1) Equation of motion

d2dt2(x1x2)=(2kmkmkm2km)(x1x2)\frac{\mathrm{d}^{2}}{\mathrm{d}t^{2}}\begin{pmatrix} x_{1}\\ x_{2} \end{pmatrix}=\begin{pmatrix} -\frac{2k}{m} & \frac{k}{m}\\ \frac{k}{m} & -\frac{2k}{m} \end{pmatrix}\begin{pmatrix} x_{1}\\ x_{2} \end{pmatrix}

Set

2km+ω2kmkm2km+ω2=0\left|\begin{matrix} -\frac{2k}{m}+\omega^{2} & \frac{k}{m}\\ \frac{k}{m} & -\frac{2k}{m}+\omega^{2} \end{matrix}\right|=0

we have

ω1=3kmω2=km\omega_{1}=\sqrt{\frac{3k}{m}}\quad \omega_{2}=\sqrt{\frac{k}{m}}

The corresponding eigenvectors are

ω1=12(11)ω2=12(11)|\omega_{1}\rangle=\frac{1}{\sqrt{2}}\begin{pmatrix} 1\\-1 \end{pmatrix}\quad |\omega_{2}\rangle=\frac{1}{\sqrt{2}}\begin{pmatrix} 1\\1 \end{pmatrix}

Then the matrix of eigenvectors is

Λ(12121212)\Lambda\equiv\begin{pmatrix} \frac{1}{\sqrt{2}} & \frac{1}{\sqrt{2}}\\ -\frac{1}{\sqrt{2}} & \frac{1}{\sqrt{2}} \end{pmatrix}

It can diagonalize the original matrix

Λ(2kmkmkm2km)Λ=(ω1200ω22)\Lambda^{\dagger}\begin{pmatrix} -\frac{2k}{m} & \frac{k}{m}\\ \frac{k}{m} & -\frac{2k}{m} \end{pmatrix}\Lambda=\begin{pmatrix} -\omega_{1}^{2} & 0\\ 0 & -\omega_{2}^{2} \end{pmatrix}

In eigenbasis,

(x¨Ix¨II)=(ω1200ω22)(xIxII){xI(t)=xI(0)cosω1txII(t)=xII(0)cosω2t()\begin{pmatrix} \ddot{x}_{I}\\ \ddot{x}_{II} \end{pmatrix}= \begin{pmatrix} -\omega_{1}^{2} & 0\\ 0 & -\omega_{2}^{2} \end{pmatrix} \begin{pmatrix} x_{I}\\ x_{II} \end{pmatrix}\quad \Rightarrow\quad \left\{ \begin{aligned} x_{I}(t)&=x_{I}(0)\cos\omega_{1}t\\ x_{II}(t)&=x_{II}(0)\cos\omega_{2}t \end{aligned}\right.\tag{$\star$}

In this problem,

1=(x1(0)x2(0))=(10)|1\rangle=\begin{pmatrix} x_{1}(0)\\ x_{2}(0) \end{pmatrix}=\begin{pmatrix} 1\\ 0 \end{pmatrix}

We first transform it into eigenbasis

(xI(t)xII(t))=Λ(x1(0)x2(0))=(12121212)(10)=(1212)\begin{pmatrix} x_{I}(t)\\ x_{II}(t) \end{pmatrix}=\Lambda^{\dagger} \begin{pmatrix} x_{1}(0)\\ x_{2}(0) \end{pmatrix}=\begin{pmatrix} \frac{1}{\sqrt{2}} & -\frac{1}{\sqrt{2}}\\ \frac{1}{\sqrt{2}} & \frac{1}{\sqrt{2}} \end{pmatrix}\begin{pmatrix} 1\\ 0 \end{pmatrix}=\begin{pmatrix} \frac{1}{\sqrt{2}}\\ \frac{1}{\sqrt{2}} \end{pmatrix}

In the eigenbasis, the state evolves according to (\star).

(xI(t)xII(t))=(12cosω1t12cosω2t)\begin{pmatrix} x_{I}(t)\\ x_{II}(t) \end{pmatrix}=\begin{pmatrix} \frac{1}{\sqrt{2}}\cos\omega_{1}t\\ \frac{1}{\sqrt{2}} \cos\omega_{2}t \end{pmatrix}

Then we transform it back to the original basis:

1(t)=(x1(t)x2(t))=Λ(xI(t)xII(t))=(12121212)(12cosω1t12cosω2t)=(12cos3kmt+12coskmt12cos3kmt+12coskmt)\begin{aligned} |1(t)\rangle&=\begin{pmatrix} x_{1}(t)\\ x_{2}(t) \end{pmatrix}=\Lambda\begin{pmatrix} x_{I}(t)\\ x_{II}(t) \end{pmatrix}\\ &=\begin{pmatrix} \frac{1}{\sqrt{2}} & \frac{1}{\sqrt{2}}\\ -\frac{1}{\sqrt{2}} & \frac{1}{\sqrt{2}} \end{pmatrix}\begin{pmatrix} \frac{1}{\sqrt{2}}\cos\omega_{1}t\\ \frac{1}{\sqrt{2}}\cos\omega_{2}t \end{pmatrix}\\ &=\begin{pmatrix} \frac{1}{2}\cos\sqrt{\frac{3k}{m}}t+\frac{1}{2}\cos\sqrt{\frac{k}{m}}t\\ -\frac{1}{2}\cos\sqrt{\frac{3k}{m}}t+\frac{1}{2}\cos\sqrt{\frac{k}{m}}t \end{pmatrix} \end{aligned}

(2) By subsitituting (x1(0)x2(0))=(10)\begin{pmatrix} x_{1}(0)\\ x_{2}(0) \end{pmatrix}=\begin{pmatrix} 1\\0 \end{pmatrix} in equation (1.8.39), we can get the same solution:

(x1(t)x2(t))=(12cos3kmt+12coskmt12cos3kmt+12coskmt)\begin{pmatrix} x_{1}(t)\\ x_{2}(t) \end{pmatrix}=\begin{pmatrix} \frac{1}{2}\cos\sqrt{\frac{3k}{m}}t+\frac{1}{2}\cos\sqrt{\frac{k}{m}}t\\ -\frac{1}{2}\cos\sqrt{\frac{3k}{m}}t+\frac{1}{2}\cos\sqrt{\frac{k}{m}}t \end{pmatrix}
 ~\tag*{$\blacksquare$}

Exercise 1.8.12 Consider once again the problem discussed in the previous example. (1) Assuming that

x¨=Ωx|\ddot{x}\rangle=\Omega|x\rangle

has a solution

x(t)=U(t)x(0)|x(t)\rangle=U(t)|x(0)\rangle

find the differential equation satisfied by U(t)U(t). Use the fact that x(0)|x(0)\rangle is arbitrary.

(2) Assuming (as is the case) that Ω\Omega and UU can be simultaneously diagonalized, solve for the elements of the matrix UU in this common basis and regain Eq. (1.8.43). Assume x˙(0)=0|\dot{x}(0)\rangle=0.

Solution. (1) Assuming that

x¨(t)=Ωx(t)|\ddot{x}(t)\rangle=\Omega|x(t)\rangle

has a solution

x(t)=U(t)x(0)|x(t)\rangle=U(t)|x(0)\rangle

Then we can get

d2dt2U(t)x(0)=ΩU(t)x(0)(d2dt2Ω)U(t)x(0)=0\begin{aligned} \frac{\mathrm{d}^{2}}{\mathrm{d}t^{2}}U(t)|x(0)\rangle&=\Omega U(t)|x(0)\rangle\\ \left(\frac{\mathrm{d}^{2}}{\mathrm{d}t^{2}}-\Omega\right)U(t)|x(0)\rangle&=0 \end{aligned}

Since x(0)|x(0)\rangle is arbitrary, we get the differential equation

d2dt2U(t)ΩU(t)=0\frac{\mathrm{d}^{2}}{\mathrm{d}t^{2}}U(t)-\Omega U(t)=0

(2) From Exercise 1.8.11, we know the Λ=(12121212)\Lambda=\left(\begin{array}{cc}\frac{1}{\sqrt{2}} & \frac{1}{\sqrt{2}} \\ -\frac{1}{\sqrt{2}} & \frac{1}{\sqrt{2}}\end{array}\right) can diagonalize Ω\Omega, and therefore, it can also diagonalize UU.

In this common basis, we have

(U¨11(t)00U¨22(t))(ω1200ω22)(U11(t)00U22(t))=0\begin{pmatrix} \ddot{U}_{11}(t) & 0 \\ 0 & \ddot{U}_{22}(t) \end{pmatrix}-\begin{pmatrix} -\omega_1^2 & 0 \\ 0 & -\omega_2^2 \end{pmatrix}\begin{pmatrix} U_{11}(t) & 0 \\ 0 & U_{22}(t) \end{pmatrix}=0
{U¨11(t)+ω12U11(t)=0U¨22(t)+ω22U22(t)=0\Rightarrow\quad\left\{\begin{aligned} \ddot{U}_{11}(t)+\omega_1^2 U_{11}(t)=0 \\ \ddot{U}_{22}(t)+\omega_2^2 U_{22}(t)=0 \end{aligned}\right.
{U11=A1cosω1t+B1sinω1tU22=A2cosω2t+B2sinω2t\Rightarrow\quad\left\{\begin{aligned} U_{11}=A_1 \cos \omega_1 t+B_1 \sin \omega_1 t \\ U_{22}=A_2 \cos \omega_2 t+B_2 \sin \omega_2 t \end{aligned}\right.

Then

x˙(0)=ddt[U(t)x(0)]t=0=(ddtU(t))t=0x(0)=0|\dot{x}(0)\rangle=\left.\frac{\mathrm{d}}{\mathrm{d} t}[U(t)|x(0)\rangle]\right|_{t=0}=\left.\left(\frac{\mathrm{d}}{\mathrm{d} t} U(t)\right)\right|_{t=0}|x(0)\rangle=0

Since x(0)|x(0)\rangle is arbitrary, we have

ddtU(t)t=0=0\left.\frac{\mathrm{d}}{\mathrm{d} t} U(t)\right|_{t=0}=0

which means

U˙11(0)=U˙22(0)=0B1=B2=0\begin{aligned} \dot{U}_{11}(0)&=\dot{U}_{22}(0)=0\\ B_1&=B_2=0 \end{aligned}

To satisfy that UU is unitary, we have

A1=A2=1A_{1}=A_{2}=1

Therefore,

U=(cosω1t00cosω2t)U=\begin{pmatrix} \cos\omega_{1}t & 0\\ 0 & \cos\omega_{2}t \end{pmatrix}

which is the same as equation (1.8.43).

 ~\tag*{$\blacksquare$}

1.9 Functions of Operators and Related Concepts

Exercise 1.9.1 We know that the series

f(x)=n=0xnf(x)=\sum_{n=0}^{\infty} x^n

may be equated to the function f(x)=(1x)1f(x)=(1-x)^{-1} if x<1|x|<1. By going to the eigenbasis, examine when the qq number power series

f(Ω)=n=0Ωnf(\Omega)=\sum_{n=0}^{\infty} \Omega^n

of a Hermitian operator Ω\Omega may be identified with (1Ω)1(1-\Omega)^{-1}.

Solution. In the eigenbasis,

Ω=(ω1ω2ωm)\Omega=\begin{pmatrix} \omega_{1} & & &\\ &\omega_{2}& &\\ & & \ddots &\\ & & &\omega_{m} \end{pmatrix}

where ωi\omega_{i} are eigenvalues.

f(Ω)=n=0Ωn=(n=0ω1nn=0ωmn)=(11ω111ωm)=11Ωf(\Omega)=\sum_{n=0}^{\infty}\Omega^{n}=\begin{pmatrix} \sum\limits_{n=0}^{\infty}\omega_{1}^{n} & &\\ & \ddots &\\ & &\sum\limits_{n=0}^{\infty}\omega_{m}^{n} \end{pmatrix}=\begin{pmatrix} \frac{1}{1-\omega_{1}} & &\\ & \ddots &\\ & &\frac{1}{1-\omega_{m}} \end{pmatrix}=\frac{1}{1-\Omega}

The third equality holds if and only if ωi<1|\omega_{i}|<1 for i=1,,mi=1,\ldots,m. Therefore, f(Ω)f(\Omega) can be defined as 11Ω\frac{1}{1-\Omega} if and only if the absolute value of each of Ω\Omega's eigenvalues is less than 11.

 ~\tag*{$\blacksquare$}

Exercise 1.9.2 If HH is a Hermitian operator, show that U=eiHU=\mathrm{e}^{\mathrm{i} H} is unitary. (Notice the analogy with cc numbers: if θ\theta is real, u=eiθu=\mathrm{e}^{\mathrm{i} \theta} is a number of unit modulus.)

Solution. Since HH is Hermitian, it satisfies

H=HH^{\dagger}=H

We can compute

U=(eiH)=eiH=eiHU^{\dagger}=(\mathrm{e}^{\mathrm{i}H})^{\dagger}=\mathrm{e}^{-\mathrm{i}H^{\dagger}}=\mathrm{e}^{-\mathrm{i}H}

Then(The second equality holds only for commuting operators.)

UU=eiHeiH=eiH+iH=1U^{\dagger}U=\mathrm{e}^{-\mathrm{i}H}\mathrm{e}^{\mathrm{i}H}=\mathrm{e}^{-\mathrm{i}H+\mathrm{i}H}=1

Therefore UU is unitary.

 ~\tag*{$\blacksquare$}

Exercise 1.9.3 For the case above, show that detU=eiTrH\det U=\mathrm{e}^{\mathrm{i}\operatorname{Tr} H}.

Solution. In the eigenbasis of HH,

U=eiH=(n=0(iϵ1)nn!n=0(iϵm)nn!)=(eiϵ1eiϵm)U=\mathrm{e}^{\mathrm{i}H}=\begin{pmatrix} \sum\limits_{n=0}^{\infty}\frac{(\mathrm{i}\epsilon_{1})^{n}}{n!} & &\\ & \ddots &\\ & &\sum\limits_{n=0}^{\infty}\frac{(\mathrm{i}\epsilon_{m})^{n}}{n!} \end{pmatrix}=\begin{pmatrix} \mathrm{e}^{\mathrm{i}\epsilon_{1}}&\\ & \ddots &\\ & &\mathrm{e}^{\mathrm{i}\epsilon_{m}} \end{pmatrix}

where ϵ1,,ϵm\epsilon_{1},\ldots,\epsilon_{m} are eigenvalues of HH, i.e.

H=(ϵ1ϵm)H=\begin{pmatrix} \epsilon_{1}&\\ & \ddots &\\ & &\epsilon_{m} \end{pmatrix}

Therefore,

detU=i=1meiϵi=eii=1mϵi=eiTrH\det U=\prod_{i=1}^{m}\mathrm{e}^{\mathrm{i}\epsilon_{i}}=\mathrm{e}^{\mathrm{i}\sum\limits_{i=1}^{m}\epsilon_{i}}=\mathrm{e}^{\mathrm{i}\operatorname{Tr} H}
 ~\tag*{$\blacksquare$}

1.10 Generalization to Infinite Dimensions

Exercise 1.10.1 Show that δ(ax)=δ(x)/a\delta(a x)=\delta(x) /|a|. [Consider δ(ax)d(ax)\int \delta(a x)\,\mathrm{d}(a x). Remember that δ(x)=δ(x)\delta(x)=\delta(-x).] Solution. Since δ(x)=δ(x)\delta(x)=\delta(-x), we have

δ(ax)=δ(ax)\delta(a x)=\delta(|a|x)

Therefore,

δ(ax)dx=δ(ax)dx=δ(ax)1ad(ax)=1aδ(ax)d(ax)=1aδ(x)dx(change ax to x)\begin{aligned} \int_{-\infty}^{\infty} \delta(a x) d x & =\int_{-\infty}^{\infty} \delta(|a| x) d x=\int_{-\infty}^{\infty} \delta(|a| x) \cdot \frac{1}{|a|} \mathrm{d}(|a| x) \\ &=\frac{1}{|a|} \int_{-\infty}^{\infty} \delta(|a| x)\,\mathrm{d}(|a| x) \quad \tag{change $|a|x$ to $x$} \\ & =\frac{1}{|a|} \int_{-\infty}^{\infty} \delta(x) \,\mathrm{d} x \end{aligned}

Thus,

δ(ax)=δ(x)/a\delta(ax)=\delta(x)/|a|
 ~\tag*{$\blacksquare$}

Exercise 1.10.2 Show that

δ(f(x))=iδ(xix)df/dxi\delta(f(x))=\sum_i \frac{\delta\left(x_i-x\right)}{\left|\mathrm{d} f / \mathrm{d} x_i\right|}

where xix_i are the zeros of f(x)f(x). Hint: Where does δ(f(x))\delta(f(x)) blow up? Expand f(x)f(x) near such points in a Taylor series, keeping the first nonzero term.

Solution. Expand f(x)f(x) around xix_{i}, where f(xi)=0f(x_{i})=0:

f(x)=f(xi)+f(xi)(xxi)+12f(xi)(xxi)2+=0+f(xi)(xxi)+O[(xxi)2]f(xi)(xxi)\begin{aligned} f(x)&=f(x_{i})+f^{\prime}(x_{i})(x-x_{i})+\frac{1}{2}f^{\prime\prime}(x_{i})(x-x_{i})^{2}+\cdots\\ &=0+f^{\prime}(x_{i})(x-x_{i})+\mathcal{O}[(x-x_{i})^{2}]\\ &\approx f^{\prime}(x_{i})(x-x_{i}) \end{aligned}

Introduce a test function g(x)g(x),\footnote{We use the Exercise 1.10.1 at the last equality.}

g(x)δ(f(x))dx=ixiϵxi+ϵg(x)δ(f(x))dx=ixiϵxi+ϵg(x)δ(dfdxx=xi(xxi))dx=ixiϵxi+ϵg(x)δ(xxi)dfdxx=xidx\begin{aligned} \int_{-\infty}^{\infty}g(x)\delta(f(x))\mathrm{d}x&=\sum_{i}\int_{x_{i}-\epsilon}^{x_{i}+\epsilon}g(x)\,\delta(f(x))\,\mathrm{d}x\\ &=\sum_{i}\int_{x_{i}-\epsilon}^{x_{i}+\epsilon} g(x)\, \delta\left(\frac{\mathrm{d}f}{\mathrm{d}x}\bigg|_{x=x_{i}}(x-x_{i})\right)\mathrm{d}x\\ &=\sum_{i}\int_{x_{i}-\epsilon}^{x_{i}+\epsilon} g(x)\, \frac{\delta(x-x_{i})}{\left|\frac{\mathrm{d}f}{\mathrm{d}x}\bigg|_{x=x_{i}}\right|}\mathrm{d}x \end{aligned}

Therefore,

δ(f(x))=iδ(xxi)f(xi)\delta(f(x))=\sum_{i}\frac{\delta(x-x_{i})}{|f^{\prime}(x_{i})|}
 ~\tag*{$\blacksquare$}

Exercise 1.10.3 Consider the theta function θ(xx)\theta(x-x^{\prime}) which vanishes if xxx-x^{\prime} is negative and equals 1 if xxx-x^{\prime} is positive. Show that δ(xx)=ddxθ(xx)\delta(x-x^{\prime})=\frac{\mathrm{d}}{\mathrm{d} x} \theta(x-x^{\prime}).

Solution. Introduce a test function\footnote{g()g(-\infty) and g()g(\infty) are finite} g(x)g(x), we have

g(x)ddxθ(xx)dx=dθ(xx)=θ(xx)g(x)θ(xx)g(x)dx=1g()0g()0g(x)dx=g()[g()g(0)]=g(0)=g(x)δ(x)dx\begin{aligned} \int_{-\infty}^{\infty}g(x)\,\frac{\mathrm{d}}{\mathrm{d}x}\theta(x-x^{\prime})\mathrm{d}x&=\int_{-\infty}^{\infty}\mathrm{d}\theta(x-x^{\prime})\\ &=\theta(x-x^{\prime})g(x)\bigg|_{-\infty}^{\infty}-\int_{-\infty}^{\infty}\theta(x-x^{\prime})g^{\prime}(x)\mathrm{d}x\\ &=1\cdot g(\infty)-0\cdot g(-\infty)-\infty_{0}^{\infty}g^{\prime}(x)\mathrm{d}x\\ &=g(\infty)-[g(\infty)-g(0)]\\ &=g(0)\\ &=\int_{-\infty}^{\infty}g(x)\delta(x)\mathrm{d}x \end{aligned}

Therefore,

ddxθ(xx)=δ(x)\frac{\mathrm{d}}{\mathrm{d}x}\theta(x-x^{\prime})=\delta(x)
 ~\tag*{$\blacksquare$}

Exercise 1.10.4 A string is displaced as follows at t=0t=0 :

ψ(x,0)=2xhL,0xL2=2hL(Lx),L2xL\begin{aligned} \psi(x, 0) & =\frac{2 x h}{L}, & & 0 \leqslant x \leqslant \frac{L}{2} \\ & =\frac{2 h}{L}(L-x), & & \frac{L}{2} \leqslant x \leqslant L \end{aligned}

Show that

ψ(x,t)=m=1sin(mπxL)cosωmt(8hπ2m2)sin(πm2)\psi(x, t)=\sum_{m=1}^{\infty} \sin \left(\frac{m \pi x}{L}\right) \cos \omega_m t \cdot\left(\frac{8 h}{\pi^2 m^2}\right) \sin \left(\frac{\pi m}{2}\right)

Solution. We start from equation (1.10.55)

ψ(t)=m=1mmψ(0)cosωmt,ωm=mπL|\psi(t)\rangle=\sum_{m=1}^{\infty}|m\rangle\langle m \mid \psi(0)\rangle \cos \omega_m t,\qquad \omega_m=\frac{m \pi}{L}

Then

ψ(x,t)=xψ(t)=in=1xmmψ(0)cosωmt\psi(x, t)=\langle x \mid \psi(t)\rangle=\sum_{i n=1}^{\infty}\langle x \mid m\rangle\langle m \mid \psi(0)\rangle \cos \omega_m t

From equation (1.10.55), we have

xm=ψm(x)=(2L)12sinmπxL\langle x \mid m\rangle=\psi_m(x)=\left(\frac{2}{L}\right)^{\frac{1}{2}} \sin \frac{m \pi x}{L}

Therefore,

mψ(0)=0L(2L)12sinmπxLψ(x,0)dx=(2L)12[0L22xhLsinmπxLdx+L2L2hL(Lx)sinmπxLdx]\begin{aligned} \langle m \mid \psi(0)\rangle & =\int_0^L\left(\frac{2}{L}\right)^{\frac{1}{2}} \sin \frac{m \pi x}{L} \cdot \psi(x, 0) \,\mathrm{d} x \\ & =\left(\frac{2}{L}\right)^{\frac{1}{2}}\left[\int_0^{\frac{L}{2}} \frac{2 x h}{L} \sin \frac{m \pi x}{L} \mathrm{d} x+\int_{\frac{L}{2}}^L \frac{2 h}{L}(L-x) \sin \frac{m \pi x}{L} \mathrm{d} x\right] \end{aligned}

where

0L22xhLsinmπxLdx=2hLLmπ0L2xdcosmπxL=2hLLmπxcosmπxL0L2+2hLLmπ0L2cosmπxLdx=2h2L2cosmπ2+2hL(Lmπ)2sinmπxL0L2=hLmπcosmπ2+2hL(Lmπ)2sinmπ2\begin{aligned} \int_0^{\frac{L}{2}} \frac{2 x h}{L} \sin \frac{m \pi x}{L} \mathrm{d} x &=-\frac{2 h}{L} \cdot \frac{L}{m \pi} \int_0^{\frac{L}{2}} x \mathrm{d} \cos \frac{m \pi x}{L} \\ & =-\left.\frac{2 h}{L} \cdot \frac{L}{m \pi} x \cos \frac{m \pi x}{L}\right|_0 ^{\frac{L}{2}}+\frac{2 h}{L} \cdot \frac{L}{m \pi} \int_0^{\frac{L}{2}} \cos \frac{m \pi x}{L} \mathrm{d} x \\ & =-\frac{2 h}{2} \cdot \frac{L}{2} \cos \frac{m \pi}{2}+\left.\frac{2 h}{L}\left(\frac{L}{m \pi}\right)^2 \sin \frac{m \pi x}{L}\right|_0 ^{\frac{L}{2}} \\ & =-\frac{h L}{m \pi} \cos \frac{m \pi}{2}+\frac{2 h}{L}\left(\frac{L}{m \pi}\right)^2 \sin \frac{m \pi}{2} \end{aligned}
L2L2hL(Lx)sinmπxLdx=L2L2hLLsinmπxLdxL2L2hxLsinmπxLdx=2hLmπcosmπxLL2L+2hLLmπL2LxdcosmπxL=2hLmπcosmπ22hLmπcosmπ+2hLLmπxcosmπxLL2L2hmπL2LcosmπLdx=2hLmπcosmπ22hL2mπcosmπ+2hL2nπcosmπhLmπcosmπ22hmπLmπsinmπxLL2L=hLmπcosmπ22hL(Lmπ)2sinmπ=0+2hL(Lmπ)2sinmπ2\begin{aligned} &\int_{\frac{L}{2}}^L \frac{2 h}{L}(L-x) \sin \frac{m \pi x}{L} \mathrm{d} x=\int_{\frac{L}{2}}^L \frac{2 h}{L} \cdot L \sin \frac{m \pi x}{L} \mathrm{d} x-\int_{\frac{L}{2}}^L \frac{2 h x}{L} \sin \frac{m \pi x}{L} \mathrm{d} x \\ & =-\left.2 h \cdot \frac{L}{m \pi} \cos \frac{m \pi x}{L}\right|_{\frac{L}{2}} ^L+\frac{2 h}{L} \cdot \frac{L}{m \pi} \int_{\frac{L}{2}}^L x \mathrm{d} \cos \frac{m \pi x}{L} \\ & =\frac{2 h L}{m \pi} \cos \frac{m \pi}{2}-\frac{2 h L}{m \pi} \cos m \pi+\left.\frac{2 h}{L} \cdot \frac{L}{m \pi} x \cos \frac{m \pi x}{L}\right|_{\frac{L}{2}} ^L-\frac{2 h}{m \pi} \int_{\frac{L}{2}}^L \cos \frac{m \pi}{L} \mathrm{d} x \\ & =\frac{2 h L}{m \pi} \cos \frac{m \pi}{2}\cancel{-\frac{2 h L}{2 m \pi} \cos m \pi}+\cancel{\frac{2 h L}{2 n \pi} \cos m \pi}-\frac{h L}{m \pi} \cos \frac{m \pi}{2}-\left.\frac{2 h}{m \pi} \cdot \frac{L}{m \pi} \sin \frac{m \pi x}{L}\right|_{\frac{L}{2}} ^L \\ & =\frac{h L}{m \pi} \cos \frac{m \pi}{2}-\underbrace{\frac{2 h}{L}\left(\frac{L}{m \pi}\right)^2 \sin m \pi}_{=0}+\frac{2 h}{L}\left(\frac{L}{m \pi}\right)^2 \sin \frac{m \pi}{2} \end{aligned}

Then

mψ(0)=(2L)124hL(Lmπ)2sinmπ2\langle m \mid \psi(0)\rangle=\left(\frac{2}{L}\right)^{\frac{1}{2}} \frac{4h}{L}\left(\frac{L}{m\pi}\right)^{2}\sin\frac{m\pi}{2}

Therefore,

ψ(x,t)=m=1(2L)12sinmπxL(2L)124hL(Lmπ)2sinmπ2cosωmt=m=1sin(mπxL)cosωmt(8hπ2m2)sin(πm2)\begin{aligned} \psi(x, t) & =\sum_{m=1}^{\infty}\left(\frac{2}{L}\right)^{\frac{1}{2}} \sin \frac{m \pi x}{L} \cdot\left(\frac{2}{L}\right)^{\frac{1}{2}} \cdot \frac{4 h}{L}\left(\frac{L}{m \pi}\right)^2 \sin \frac{m \pi}{2} \cos \omega_{m} t \\ & =\sum_{m=1}^{\infty} \sin \left(\frac{m \pi x}{L}\right) \cos \omega_{m} t \cdot\left(\frac{8 h}{\pi^2 m^2}\right) \sin \left(\frac{\pi m}{2}\right) \end{aligned}
 ~\tag*{$\blacksquare$}